0% found this document useful (0 votes)
475 views93 pages

Computer Oriented Statistical Methods CH 1to 3

Uploaded by

Anshul Saxena
Copyright
© © All Rights Reserved
We take content rights seriously. If you suspect this is your content, claim it here.
Available Formats
Download as PDF, TXT or read online on Scribd
0% found this document useful (0 votes)
475 views93 pages

Computer Oriented Statistical Methods CH 1to 3

Uploaded by

Anshul Saxena
Copyright
© © All Rights Reserved
We take content rights seriously. If you suspect this is your content, claim it here.
Available Formats
Download as PDF, TXT or read online on Scribd
You are on page 1/ 93

Elements of

Cohy (Orientietdl
Scaistical
S slethveréls
For B.C.A

JEEVANSONS PUBLICATIONS
SYLLABUS
(K.U., Kurukshetra)
PAPER: B.CA. 245 : COMPUTER-ORIENTED STATISTICAL METHODS
FOURTH SEMESTER
: External Marks:80
Maximum Maržs 100
Time Allored:3Hours Internal Marhs: 20
Note. Erzminereill be required to set nine questions in al. First question will be compulsory,
consisting of objective type/short-ansuer type questions covering the entire syllabus. In
addition to that eight more questions will be set, tuo questions from each unit. A
candidate willbe required to answer fire questions in all, selecting one question from
eac: urzit in addition to compulsory question No. l.All questions will carry equal marks.

Unit -I
Basic Statistics :Preparing Frequency Distribution Table and Cumulative Frequency, Measure of
Central Tendency, Types; Arithmetic Mean, Geometric Mean, Harmonic Mean, Median, Mode.
Measure of Dispersion:Range, Quartile Deviation, Mean Deviation, Coeficient ofblean
Deviation,
Standard Deviation.
Moments: Moments about mean, loments about any point, oment about origin, Momentmean. about
mean in terms of moment about any point, Moment about any point in terms of moment about

Unit-II
Probability Distribution Random variable, Discrete Random and Continuous Random variable,
:

Probability distribution of a Random Variable, Mathematical Expectation.


Poisson and
Types : Binomial, Poisson, Normal Distribution, Mean and variance of Binomial,
Nornal Distribution.
Correlation : Introduction, Types, Properties, Methods of Correlation; Kari
Pearson's Coefñcient of
error.
Correlation, Rank Correlation and Conaurrent Deviation method, Probable
Unit - II
Regression : Introduction, Aim of Regression Analysis, Types of Regression
Analysis, Lines of
Regression, Properties of Regression Coefficient and Regression Lines, Comparison with
Correlation.
curve.
Curve fitting: Straight line, Parabolic curve, Geometric curre and Exponential
Baye's Theorem in Decision making, Forecasting Techniques.
Unit-IV
Sample introduction, Sampling Meaning, methods of Sampling, Statistical Inference :Test
: of
error, one Tailed and
Hypothesis, Types ofhypothesis, Procedure of hypothesis testing, Type I and Type I no. sucess
two tailed Test, Types of test of Significance; Test of significance of Attribute-Test of of and
test of proportion of success, Test of significance for large samples. Test of significance for single mean
and Difference of mean, Test of significance for small samples (t-test) - test the significance between the
mean of a random sample, between the mean of two independent samples.
Chi square Test, ANOVA: Meaning, Assumptions, One way classification, ANOVA Table for One
Way Classified Data.
CONTENTS

Pages
Chapter
i: 1.1. -1.16
. eee

Basic Statistics
,i21 2,46
Measure of Central Tendency
:N2is
ye
tst:
2.
3.1 3.26
*rt
Measure ofDispersion
3.

4. Moments and Moment GeneratingFunction


.... 4.1 - 4.22

5. Probability Distributions
5.1 - 5.56
tf.rtiti';l-ihii
unet Correlation 6.1- 6.43

n Regression
71727
.i. ittis8.1 -8.24
8. Statistical Inference
cenet
9. Chi-Square Test
9.1 - 9.18

-10.11
10. Curve Fitting*
Analysis of Variance (ANOVA) 11.1- 11,13
11
eee

Making Forecasting
&

12. Baye's Theorem in Decision *. t! 121- 12.22


Techniques*

Question Paper ()-(v)

For K.U. :Chapter 10 and 12 are included in Unit IIl the syllabus.
of
1 Basic Statistics

1.1.
Introduction
or
Tho rord statistics has been derived from the Latin word 'status' the Italian word
were interested in
ictetista' both meaning the same a political state. In olden times, the rulers
erelfare of their states and kept records of persons who paid taxes, record of the strength
of
armr ete. Thus arose the need of collecting the various data known as statistical data. In India,
re fnd such records of statistical data being kept since the times of Emperor Akbar in Abul
Baznl's Ain-i-Akbari
. Ronald A.Fisher (1890– 1962), an English mathematician has been called
vast contribution to
the real giant in the development of the theory of statistics. Due tohis
statistics, Prof. Fisher is known as the Father of Statistics'.
We find now-a-days that statistics has its application in many branches ofknowledge such
as: Economics, Business, Industry,Mathematics, Biology, Astronomy, Psychology, Education
and Medical Sciences.
1.2. Population
term
the word population we mean, collection of individuals. The or
In statistics, by
synonyms. This collection can be finite
group, aggregate or totality of objects are its
infinite. Universe is the older term for the word population.

Illustrations.
of your class in the school is a population
1. The population of the age of allthe students
consisting of finite number of individuals.
2. If we count the number of children born in the
year 2008 and write down 0 for
more than 6 pounds,
children weighing less than 6 pounds and T for children weighing
then the collection of
0'
and T forma population.
There are some populations which consists large number
of
ofindividuals. Such populations
for all practical purposes are regarded
as infinite population.
a population of infinite individuals.
3. The set of rational numbers lying between 2 and 3 is

4. The example ofleaves on tree, the number


a of hair on our body ete., are also examples
of infinite population.
1.3. Sample
we
Sample is that part of
thepopulation which is selected for investigation. From sample
try to investigate the properties of the population from which the sample has been selected.
Sample can also be defined as :
a
Any finite set of items drawn from a population for investigation is sample.
MATHEMATICS FOR B.C.A.
1,2
characteristic
practically it is impossible to doterminea single
ltis an established fact that population, since 100% inspection is not possible (neither
of each and
every member of the
causcs namely administrative, financial or time
multiplicity of a kettle have boiled or
necessary). It may be due to the rice boiling in
if we want to know whether serve our purpose.
factor etc. For example, spoon is sufficient to
a few rice taken out by
not, then only a sample of necessary.
Checking of each grain ofrice is
neither practical nor means of a
collected about the population by
the information sampling or some other
It should be noted that due to defective
are not accurate. This might be size (the number of
individuals)
sample sometimes can reduced by incrensing the
reasons. The samnpling errors be observations,
ifn < 30 where n is the number of
sample is considered small
in the sample. The
considered large.
and ifn > 30, the sample is
1.3.1. Smpling on the basis of a sample
process of learning about the population
Sampling is simply the of everyindividual of the
population, only
sampling technique, instead
drawn from it. Thus in conclusions are drawn. The process of sampling involves
a part i.e., sample is studied and the
three steps, viz., sample.
(2) Collecting the information about the
(1) Selecting the sample.

(3) Making an inference about the population.

1.3.2. Purpose of Sampling


own sake. The basic objective of its study is to get knowledge
A sample is not studied for its
been taken.
about the population from which the sample has
1.3.3. Methods of Sampling
(a) Simple or Random Sampling
every individual of the
Random sampling refers to the sampling technique in which
a sample. This selection is free from
population is given an equal chance of being selected in
personal bias or prejudice.
or
Lottery method is a popular example of simple random sampling.
(6) Non-Random Sampling
A sampling which is not random is called non-random sampling.

Judgement sampling is the best example of non-random sampling. In this type of


sampling, the taste of judges is one of the criteria.
L.3.4. Individual. An individual is one member ofthe population.
1.4. Variable or Variate
A quantity which can vary from one individual to another is called a variable or variate
or in other words any character capable of taking different values is called a variate. For example,
height of students in a class, marks obtained by the students in an examination etc. are variables
or variates.
STATISTIC
BASIC 1.3

variablewhich ean take any numerical value within a certoin


A

rongeis calledl continuous


Variable, Por example, height or weight of a child at various ages a A

mriable uhich cannol take all possible values in a range is


is continuous variable.
called discontinuous or discrete
variable. For example, number of rooms in a house can values
take only the integral
3,4, 5, etc. The quantities which are measurable in nature like height, weight, marks, income
etc, are cnlled quantitative variables and quantities which are non measurable in nature
like beauty, honesty ete. are called qualitative variables.

L.5. Primary, Secondary and Raw Data


The word data means information or a set of given fucts.
The data collected by the investigator himself witha specific objective or purpose is known
B Primary data. Such data are original in nature.

If the investigator, due to lack of time and other resources, uses the data collected by
Ome other agency, then sucha data is known as secondary data. In other words, when the
data collected by some, is put touse by another or data collected for one purp0se is applied to
Another, then such data is called secondary data.

The values of a variate collected in an arbitrary order in which they occur (i.e., original
form) are called raw data. For example, marks of 35 students (out of total marks 50) according
to their rollnumbers are:
45 30 21 12 15 5 2 7 18

27 40 35 21 15 7 2 7 21 35

40 7 24 18 45 12 27 21 5

27 45 12 21
This is an exampleof raw data.
1.6. Array
The arrangement of data in ascending or descending order is called an array.
Now we shall arrange the data given in Art.1.5 in ascending order. The arrangement thus
becomes:
2 3 5 5 5 7 7

7 12 12 12 15 15 18 18 21 21

21 21 21 24 27 27 27 30 35 35

40 40 45 45 45
which is an array.
1.7. Frequency
Thenumber of times a variate is repeated is called its frequency. For example, if the
varíate x take the values 2, 2,3, 3, 5, 5, 5,then the variate 2 is repeated 2 times, 3is repeated
2times, 5 is repeated 3 times, We say
that frequency of 2 is 2, frequency of 3 is 2, and
frequency of 5 is 12'. The sum of frequencies is called the total frequency and is generally
denoted by N.
MATHEMATICS FOR B.C.A.
1.4

1.8. Frequency Distribution of each


is a tabular arrangement of data showing the frequency
A frequency distribution
variate.
1.8.1. Ungrouped Frequency
Distribution or Discrete Series :

in the form of a table (see table 1)


in Art.1.5
Let us write the data given obtained by each
obtained from the raw data by tally method. The marks
This table can be belongs. The fifth
a vertical line against the variate to which it
are represented by an oblique line crossing the
candidate marked by
by a vertical line but it is
observation is not shown some space after first five
observation is again marked by a vertical line leaving of each variate.
first four. Sixth observations, we can quickly write the frequency
and soon. After marking all the 1
Table
Tally Marks No. of Students
Marks (Frequency)
(Variate)
2
2
3
5
4
7

12
2
15
18
2
5
21
1
24
27
30 1

35 2

40 2
45
Total frequency N=35
The representation of data as in table l is called frequency distribution of an ungrouped
dataor discrete series.
1.8.2. Grouped Frequency Distribution
If the above table is still lengthy or incomprehensible, we can make it simpler by grouping
the data as follows:
STATISTICs 1.5
BASIC

"I'able2
Marks Tally Marka No, of AtudentA
(Class - interval) (Frequency)
1-5 7
6-10
11- 15
16 -20 2
21 - 25
26 -30 4
31-35 2
36 –40 2
41- 45
46 – 50
N= 35
In this table, we have condensed the data into classes. Class
1 -
5 means marks obtained
between 1 and 5. Here 1 is the lower limit of the class and is the
upper limit of the class.
upper limit. The width of each
Similarly, in class 6-10; 6 is the lower limit and 10 is the
interval is 5.
The number of observations fallingin a perticular class is called the frequency
of that class.
The representation of data as in table 2 is known as discrete grouped frequency
distribution. we can not
If we want to prepare a data of heights ofstudents in a class of your school then
willbe no student between
preparea tableas we have done in table 2. We cannot say that there
no student has secured marks between 5
certain interval, as we have assumed in table that
2
so
we divide the marks in groups -5, 5-10, 10– –15 and
0

and 6. 10 and 11 etc. In such a case


on. The boundary values are put in higher group i.e., 10 is to be put in
the group 10 15 and not
:

in 5- 10. The frequency distribution thus


becomes
Table 3
Tally Marks No. of students
Marks (Frequency)
(Class - interval)
4
0-5
7
5- 10 3
10- 15
4
15 -20
6
20 -25
3
25-30 1
30-35 2
35-40
2
40 – 45
45 -50
N=35
MATHEMATICS POR B.C,A.
1.6

distribution ns ahown in table 3 is known as continuous grouped


The grouped frcquency
frequency distribution, :
we have following two types of frequency distributions
Thus
: If
tho upper limit ofa class is also included in
(1) InelusiveForm
(Discontinuous) an
case 2, then such a form of frequency distribution is called
as in table
that class in the
inclusive form. a way that the
(Continuous) :
If tho classes aroforned in such case of
(2) Exclusive Form
a excluded in this clas8 and is includod in the next class as in the
upper limit of class is
distribution is called an exclusive form.
table 3, then such a form of frequency
1.9. Some Important Definltlons
are
the groups into which the
1. Class Intervals and Class-limits.
Class intervals
raw data is condensed. Theclass-limits are the lowest and
highest values that can be included
are included, one is known as lower limit (L,) and the
in the class. In one class, two limits
known as upper limit (L,). For example, in the class 10- 20; 10 is the lower limit and
other is
20 is the upper limit.
can be made with the help of following
The estimation of the number of class-intervals
formula:
= Maximum Value - Minimum Value
Number of class-intervals Size of Interval

Magnitude of the Class-Interval or Size of Interval. The difference between the


2.
or
upper limit and lower limit of a class is called magnitude of class interval or size of interval
-
width of interval. For example, in the class 15 25, the magnitude of class-interval
is (25- 15) i.e., 10. Thus, we have the following formula
i =Ly- L

where i= Magnitude of class-interval


L, = Upper Limit and L, = Lower Limit.
3.Class Mark or Mid-Point or Mid-Valueof Class.The centre of the limits of a class

is called class mark or mid-point or mid-value of a class. It is the value lying half-way between
the lower and upper class limits of a class-interval. Class mark or mid-point of a class is ascertained
as follows:

Lower limit + Upper limit


Class mark of a class =
2
+ 25
For example, if the class interval is 15 – 25, then its class mark =
15
2
40
-=20.
2
4. Class Frequency. The number of observations corresponding to a particular class is
called the frequency of that class.
STATISTICs 1.7
BASIC
:
Consider the following table

Marks Obtained No. of Students

0- 10 22

10-20 14

20-30 19

30– 40 32

40- 50 13

Total 100

In theabove table, the frequency class interval 10 -20 is 14 which implies that there
of

eme 14
students who have obtained marks between 10 and 20. If
we
add together the frequencies
case.
afall the individual classes, we obtain the total frequency which is 100 in this
a
Cumulative Frequency. The cumulative frequency corresponding to class is the
5.
Sum all frequencies upto and including that class.
of

are called less


If the data is arranged in ascending order, the cumulative frequencies
cumulative
than cumulative frequencies and if the data is arranged in descending order, the
frequencies are called more than cumulative frequencies.
1.10. Types of Continuous Series
Continuous series may be of the following types:
1. Exclusive Series
2. Inclusive Series
3. Simple (Non-cumulative) Series
4. Cumulative Series
5. Open ended Series
(1) Exclusive Series. When the class-intervals are
so fixed that the upper limit of one
an exclusive
class-interval is the lower limit of the next class-interval, then the series is called
series. Following is an example of exclusive series :

Class-interval Frequency

0-5
10 12
5-
10 - 15 20

15 - 20 16

20 - 25 11

It is clear that the exclusive series ensures continuity of data since the upper limit of one
class is the lower limit of the next class.
MATHEMATICS FOR B.C.A.
18
a
In inclusive series, both the lower límit and the upper limit of
(2)Inclusive Series. of one
are included in that class itself. In other words, the upper limit
class-interval
next clagg-interval, Following is an example of
class-interval is not the lower limnit of the
inclusive series:

Marks obtained No. of 8tudente


20
50- 59
15
60- 69
18
70- 79
80- 89 30
90- 99 17

Total 100

Conversion of Inclusive Series into Exclusive Series


we employ the following
To convert an inclusive series into an exclusive series,
two steps:
a
1. First, we find the difference between the upper limit of class interval and the lower
limit of the next class interval.
2. Halfof this difference is added to the upper limit of each class interval and remaining
half is deducted from the lower limit of the each class interval.
Using these two steps, the inclusive series can be converted into an exclusive series.
In the example given in the inclusive series above, we observe that difference between
upper limit of a class interval and lower limit of next class interval is 1.
The half of this difference i.e., 0-5 is added to upper limit of each class interval and other
half i.e., 0-5 is deducted from lower limit of each class interval. Thus the inclusive series is
converted into exclusive series, which is as given below:

Marks obtained No. ofStudents

49-5 - 59-5 20
59-5 - 69-5 l5
69-579-5 18
79-5 - 89-5 30
89-5 -99-5 17

Total 100

Remark: Magnitude of class-interval in both the cases will be equal.


STATISTICs 1.9

sC or Non-cumulative Frequency Series. In a simple series, the frequency


(S) Simple
sgAinst each
class-interval is shown separately and individually. Following is an example of
sinple
series:
Class Frequency - Simple
0-5 10
5 - 10 15

10 - 15 25
15- 20 30

20- 25 20

Cumulative Frequency Series. In a cumulative frequency series, the frequencies


(A)

nrogressively totalled and aggregates are shown. Following is


an example of cumulative
&re
series :
Value Frequency-Cumulative
0-5 10

0- 10 25

0- 15 50

0- 20 80

0- 25 100

Conversion of Simple Frequency Series intoCumulative Frequency Series


frequency series in the following
A simple frequency series can be converted into cumulative
Maner:
:
Consider a simple frequency series given below
Frequency - Simple
Class-interval
10
0-5
15
5– 10
10 - 15 25

15 - 20 30

20 - 25 20

The above given simple frequency series can be converted to cumulative


frequency series
in the following ways :
MATHEMATICS FOR B,C.A.
1.10
:
(a) Less than Cumulative Frequency Table
Frequency-Cumulative
Value
10
Less than 5
= 25
Less than 10 10 + 15
= 50
Less than 15 10 + 15 + 25
+ - 80
Less than 20 10 + 16 + 25 30
+
20 = 100
10+ 15 + 25 +30
Less than 25

(6) More than Cumulative Frequency Table:


Frequency-Cumulative
Value
+
30 + 20 = 100
More than 0 10 + 15 +25
= 90
More than 5 15 + 25 + 30 + 20
= 75
More than 10 25 + 30 + 20
= 50
More than 15 30 + 20
20
More than 20
Frequency Series
Conversion of Cumulative Frequency Series into Simple
following
frequency series can be converted into simple frequency series in the
A cumulative
manner:
(1)

Value Frequency-Cumulative Class-interval Frequency-Simple


10 0- 10 15
Blow 15

Below 20 20 10 - 20 20 -
15=5
= 40
Below 30 60 20-30 60- 20
Below 40 85 30 – 40 85 - 60 = 25
Below 50 100 40- 50 100 – 85 = 15

(2)

Value Frequency-Cumulative Class-interval Frequency-Simple


More than 0 100 0-5 100 – 90 = 10
More than 5 90 5- 10 90- 75 =15
|More than 10 75 10 - 15 75- 50 = 25
15 -
More than 15 50 20A 50- 20 =30
|More than 20 20 20 - 25 20
1.11
T7STICS
STA
BASIC

as more than, above, below,


Remark: we given any of the
are
words such less than,
any. series
etc. before allthe limits of
class-intervals then it
1 Iin exceeding, not exceeding,
under, upto, series,
frequency
isa cumulative frequencies, not of items.
remembered that cumulation is
of

be
2 Itisto are those series in which the lower-limit of
a Onen ended Series. Open ended series are not given. It is clear
class-interval and the upper-limit the last class-interval
of

thefirst below :
fonh the
examples given
(1) 2)

Value Value

Less than 5 Below 5

5-10 5-15
15 - 30
10- 15

15 - 20 30- 50
20 - 25 30 -75
75and above
25 and above

the above tables both the first and the


In last classes are open-end classes.
so classes
In case of table (1), the difference of
class-intervals is the same, the open-end
class-intervals is not the
can be adjusted accordingly. But in
case of table (2), the difference of
and last class-interval would be
75-105.
same. So here the first class-interval would be 0-5
SOLVED EXAMPLES

it
distribution from the following data arranging
Example 1. Form a discrete frequency
ascending order (ü) descending order.
in ()
15 11 15
7 9 5 9
7 5
15 19 15 19
13 19 17 17
13 11
Frequency Distribution (Ascending order)
Solution. (i) Formation of Discrete

Tally bars
Frequency (f)
Variable (X)

2
5

2
7
3

2
11
2
13
4
15
2
17
3
19

Total 20
1.12 MATHEMATICS FOR B.C.A.

(ii) Formation of Discrete Frequency Distribution (Descending order)


Variable (X) Tally bars Frequency (f)
==
19
17 2

15 4
13 2

11 2

7 2

6 2

Total 20

Example 2. The following is a record of marks of 50 students. Represent the data in the
form of a frequency distribution, taking the lowest class as 40-49:
60 65 70 74 80 75 90 49 59 69
40 50 60 45 55 65 79 89 99 72
74 76 44 48 53 60 64 63 84 86
92 93 96 99 89. 69 44 60 70 90
72 80 91 53 54 58 63 68 69 78

Solution. Formation of Frequency Distribution


Marks Tally Bars No. of Students ()
40 – 49 6

50- 59,. 7

60-69i NN
13
70 - 79 10
80 - 89 6
90 - 99 co

Total 50
Example 3. Put the following data relating to marks obtained by 40 students in the form of
frequency distribution with a difference of 10in :(i) Exclusive Series (ii) Inclusive Series.
Marks Obtained (Out of100)
53 15 48 40 42 46: 62 75 96 86
73 38 27 20 66 97 19 55 58 77
78 62 71 79 93 90 88 06 23 07
04 54 60 52 67 65 78 73 78 36
STATISTICS 1.13
BASIC

Solution. Here the


smallest value is 4 and largest value is 97. The class-intervals should
may be included. The magnitude of class interval
way so that all the values
formed in sucha 0 - 10,
20 and so on and

be case of exclusive forn, the class-intervals would be 10
be 10. In - so on.
istocase ofinclusive form, the class-intervals would be 0-9, 10 19 and
in Series)
Eormation of Frequency Distribution (Exclusive
Tally Bars No. of students (f)
Marks obtained

0-10
10 20- 2

20 -30
2
30 -40

40 - 50 4

50 - 60 5

60 - 70 6

9
70 – 80
2
80- 90
90 - 100 4

Total 40
Series)
(iü) Formation of Frequency Distribution (Inclusive
Tally Bars No. of students ()
Marks obtained
3
0-9
10 - 19 2

20 - 29
2
30 - 39
4
40 – 49
50 - 59 5

6
60-69
7o79
80 - 89 2

90-99
Total 40
MATHEMATICS FOR B.C.A.
1.14

ascending order in inclusive form from the following


Example 4. Make a frequency 3table in :
data, selecting a class-in terval of units each
10 11 13
12 14 6 9 7
5
22 16 19 21
9 8 10 7 16 17

9 13 17 16
12 13 6 7
20 18
inclusive form (ascending order)
Solution, Formation offrequency distribution in
Frequency
Class-intervals Tally bars
6
5-7
7
8- 10
6
11- 13
4
14- 16
19 4
17-
20 - 22 3

Total 30
:

Example 5. Form an ordinary frequency table from the following table

Height (in ft) No. of trees Height (in ft) No. of trees

Below 7 26 Below 35 216


Below 14 57 Below 42 287
Below 21 92 Below 49 341
Below 28 134 Below 56 360

Solution. Here we are given less than series. The width of the class interval is to be
taken as 7. Thenumber of trees having height between 0-7 ft are 26, between 7-14 ft are
57-26= 21 and so on.
Hence the required ordinary frequency distribution table is :
Height (in ft) No. of trees Height (in ft) No. of trees
0-7 26 28-35 216– 134 = 82
7-14 57-26 =31 35-42 287-216 =71
14-21 92- 57=35 42- 49 341- 287 = 54
21-28 134- 92 = 42 49- 56 360-341 = 19
STATISTICS 1.15

cExample 6. Given below is grouped frequency distribution


a of marks. Convert this frequency
more than form.
i). less than form (ii)
tableinto
Marks : 0- 10 10-20 20-30 30 -40 40-50

students : 6 7 6 4
No. of
=
Here the number of
students securing marks less than 10 5; less than
Solution.(i)
113; less than 30
= 5
+8+7= 20 and so on.
20=5+8=
is
Thus, less than form of frequency distribution
Marks No. of students

Less than 10

Less than 20 13

Less than 30 20

Less than 40 26

Less than 50 30

Remark:
we at once know that the number of students
The advantage of such a table is that
30are 20 etc.
getting marks less than 20are 13, less than
of students = 30.
(iü)The total number
0 is 30: more than 10 is 30
- 5 = 25;
The number of students securing marks móre than
on.
more
than 20 is 25- 8 = 17 and so
is
Thus, more than form of frequency distribution

Marks No. of students

More than 0 30it


More than 10 25
17
More than 20
10
More than 30
4
More than 40

Remark:
or more marks
From this table we at once know that number of students getting 20
are 17 and number of students getting 30
or more
marks are 10 etc.
1.16
MATHEMATICg FOR B.C.A.

1.11Relatlve Frequency
a freguencv table is the number of variates
We know that the frequency of anvclass in
falling in that class. Sometimes this number (i.e. the freauencv) is expressed as a fraction of the
totalfrequency and is called the relative frequency of that class.
= Class frequency
i.e., Relative frequency Total frequency
each frequency
relative frequency of a frequency distribution is obtained by expressing
The
as a percentage of total frequency.
= Class
frequency -100%
Relative frequency (%)
i.e., Total frequency
distributions
frequency distributions are used to compare two or more frequency
us consider the
The relative same freguency distribution. For example, let
or two or more items of the following
obtained by students in class in the
a
distribution of the marks
relative frequency
:
data Relative
Frequency Relative
Class interval frequency (%)
(No. of students) frequency
6 = 24%
0-24
20 - 40 6 25
12-= 0-48 48%
40–60 12
25
4 = 0-16 16%
60-80 4
25
3 = 12%
3 0-12
80- 100 25
1-00 100
Total 25
may be observed that the sum of all relative frequencies is 1-00 and that of relative
It
frequency (%) is 100.
marks
above example, we can easily compare the number of students securing the
Inthe
are 48% students who have secured marks between 40 -60 and
40– 60 and 60 -80. (As there 60 - 80). In the same manner we can also
16% students who haye secured marks between
of two classes A and
compare two frequency distributions, For example, the marks of students
Bare given in the following table.
Relative frequency table
Relative frequency %
Class marks No. of students No. of students
of class A of Class B Class A Class B
O-10 5 5

10-20 25 12 25 20
20-30 20 16 20 26-66
30 -40 35 17 35 28-33
4050 15 20
12 E15
Total 100 60 100 99-99
STATISTICS 1.17.
BASIC
is clear that in class A, 20% students have obtained marks between
it
Fromthe.above table
class B. 26-66% students have obtained marks between 20- 30.
N-301whereasin
means of the relative
.ehnuld be noted that for comparing two frequency distributions by
as wvell as the division ofthe values of the variables into classes must
frequencics, the variables
same for both distributions.
hethe EXERCISE 1.1

ballowing data gives the number of children in 40 families. Represent it in the form of
frequency distribution.
3 1 4 4 4 6 5 2 4
4

2 3 3 6 2 2 2
2
2 1 2 0 2 1 1
5 2
4 1 2 2 1 2
5
a Make a continuous frequency table of
wages with class size of rupees 2 each, from the
following data of daily wages received by 30 labourers in a certain factory. Daily wages in
rupees are :

22, 13, 15, 24, 12, 23, 14, 20, 17, 21, 22
14, 16, 16, 14,
12, 21, 20, 17, 18, 19, 23.
18. 18, 19, 20, 17, 11, 16, 15,
scores in an examination are 10, 30, 50, 70, 90.
The class marks (mid values of class) of
3.
Determine the class size and the class intervals.
Thirty 16 years old boys were tested to find their pulse
rate. The following figures were
4.
obtained for the number of beats per minute.
70, 74, 53, 57, 62, 71, 58, 68, 75, 79,
55, 72, 70, 66, 74,
60, 72, 56.
68, 63, 59, 54, 51, 61, 66, 78, 73, 59, 52, 66,

Using the class-intervals 51 - 55, 56 – 60, etc. of equal width, prepare frequency table.
a
:

5. Form an ordinary frequency table from the following


table
Marks obtained No. of students
3
Below 10
Below 20 8

Below 30 17

Below 40 20

Below 50 22
:

6. Form an ordinary frequency table from the following table


obtained No. of students
Marks
Above 0 30

Above 10 26

Above 20 21

Above 30 14

Above 40 10

Above 50
NAIEMATICS FOR HCA
1.18
ii) more than form.
o) lenn than form
7. (onvert the folloring frequeney table into R0. )
.60
10-) 0 B0
Merk
9 :
of students
No.
reoueney distrilution of the following datai 20 - 25 26 - 30
8. Pnd he relative 10 10 -- 16
-
16 20
6

(lass : 193
266
172 383
Frrquency :
ANSWERS

No, of childre)
1. 13
No, of familios:
16 16 -- 17 17- 19 19
-
2121-23 23-25 Total
Wages :
l1- 13|13- 4 30
2. :
No, of labourers
80, 80 - 100
Size of class = 20: 0- 20, 20 -40, 40-60, 60-
3.
71-75 76 - 80
G6 -- 60 61-65 66-70
No. of bents
:

51-55 7
ma
3 7
No, of boys
:
5

- 30 30 -40 40 - 50
10- 20 20
Marks: 0-10
3 2
5. 5 9
No. of
students:

20 -30 30 -40 40- 50 50 - 60


Murks
:
0-10 10 -20
6. :
7 5 5
No, of students

20 30 40 50 60
Marks (Less than ): 10
7. ()| S0
No. of students
:
12 27 57 75

Murks ( Mure than ): 10 20 30 40 50

No. of :
80 77 68 53 23 5
students w

Cluss :
0-- 5 5
10 10 - 15 15 - 20 20-25 25 - 30
8.
Relative frequency (%) 8-33 18-56 23-55 12-84 27-37 935
2 Measure of Central Tendency

2.1. Introduction
A
single value of the variable representing the entire data, which describes the
characteristics of the data is called an average of the data. Since the average tends to lie
centrally with the values of the variable, arranged according to nagnitude, it is also called
measure of central tendency or measure of location.
A measure ofcentral tendency or average helps us in knowing the average character of the
data under investigation, by a single quantity. This is the quantity around which the whole data
tends to cluster. Under different types of situations, data tends to cluster on different individuals
having nature of different type. Thus a single type of average is not suitable under all types of
situations. Consequently, there are five measures of central tendency which are commonly
used.
2.2.Types of Measures of Central Tendency
fa) Arithmetic Mean (A.M.)
b) Geometric Mean (G.M.)
c) Harmonic Mean (H.M.)
ful) Median
te) Mode
2.3. Characteristics for an ldeal Measure of Central Tendency
() It should be rigidly defined i.e., it must, have one and only one interpretation.
(ü) It should be based on all observations.
(üi) It should be easy to understand and simple in calculation.
Extreme values should not have much effect.
(iv)
It should be least affected by sample fluctuations.
(u)
2.4. Arithmetic Mean (A.M.)
Arithmetic Mean is the most important and widely used measure of central tendency. It is
generally referred as “mean" alone.
"The Arithmetic mean is the number which is obtained by adding the vàlues of all the
variables ofa series and dividing the total by the number of items."
2.4.1. Ungrouped Data (Individual Series)
The arithmetic mean of n quantities *1, tzy
.., x, is denoted by and is given by

i=1
Or
123
MATHEMATICS FOR B.C,A,
2.2

2.4.2. Grouped Data (Discrete Series) with frequencies f f n respectively


values x,, ,X,
IF thevariate x takes the
a,
(i.e., frequcncy of is f). then

A.M, =

il

iel where N-XA


N
distribution, we consider the mid-value ofclass
interval s the variate.
1. Ina grouped fequency
Note.
can also use E
f and Ef respectively.
2. For i;and )f,we
-1

2.5. Weighted Arithmetic


Mean
given to the variates x,, tg,.., x,, then
W,, denote the weights
(importance)
If w,, wgy..,
+.. +
W, n Ew;x
Ew;
W
+ W, t.... + W,
and is called the weighted arithmetic mean.
SOLVED EXAMPLES
of a
(A.M.) of the marks obtained by 9
students
Find the Arithmetic Mean
Example l.
class given below
52, 40, 70, 43, 75, 40, 48, 35, 65.

Solution. Let be the Arithmetic Mean, then


+ +
52 + 40+ 70 + 43 + 75 + 40 48 35 65
+

468 = 52.
9
:

Example 2. Find the Arithmetic Mean of the following data


1
|

3 4 5 6 7 9 10
Roll No.:
Marks in Maths: 52 75 40 70 52 43 40 65 35 48

Solution. Let the variable marks in Maths be denpted by x.


+ 48
52 + 75 + 40 + 70 + 52 + 43 + 40 + 65 +35
A.M. (z ) =
10
520
52,
10
NMEASURE OF CENTRAL TENDENCY 2,3

Example 3. Tle mean marks of100 students was found to be 40. Later on, it was discovered
that score of 53 was misread as 83. Find the correct meun.
a
Solution, Here numbor of students = 100
Incorrect mean = 10
Incorrect marks = 83
Correct marks = 53
Let the variable marks bo denoted by x

x incorrect Ex
Incorrect
100
E

i.e. 40 = incorrcct
100
Incorrect Ex = 40 x 100 = 4000.
,
Let the correct A.M. be obtained by using correct Ex
correct Ex = incorrect Ex- incorrect x+ correct x

= 4000- 83 + 53 = 3970
3970 = =
Correct mean () 39-7.
100
Example 4. Calculate the A.M. for the following data :
Marks obtained: 36 42 46 55 63 72
No. of
students: 4 5 9 10
:
Solution. Calculation ofA.M.
Marks (x) No. of students (f) fx
36 4 144
42 5 210
46 9 414
55 10 550
63 8 504
72 4 288
N= 40 Efx = 2110

Efx 2110 = 52.75.


N 40
:
Example 5. Calculate the A.M. from the following datu
Marks :
0- 10 10- 20 20 -30 30 - 40 40 -- 50 50 - 6O
:
No. of students 7 15 3 3
123
MATHEMATICS FOR B.CA,
2.4

Solution, Calculation of A.
M.:
No. of students () fx
Marks Mid points of classes (*) 20
4
0 10 7 105
10-20 16 375
t 16
20 -30 25 280
8
30 40 35 135
40- 50 45 165
55
50--60
N=40 Efx = 1080

Efx 1080 - 27.


A.M. (E) = 40
N frequencies are
numbers whose
Example 6. Find the arithmetic mean of first n natural
numbers.
equal to corresponding ....,n and their respective frequencies are
are 1, 2, 3,
Solution. Firstn natural numbers
1,2, 3, .., n.
n(n+1)(2n +1)
n.n En? 6
1.1+2.2 + 3.3 +....
+

A.M, = En n(n+1)
1+2 +3+.... +n 2

+ 1) (2n + 1) 2 2n + 1
n (n
6n (n + 1) 3

the Mean (Shift of origin and change of


scale)
2.6. Step Deviation Method for Finding
We observe that in cásés where the values of themean
variable (x) and their respective
by the usual formulae becomes
frequencies (f) are large, the calculation the arithmetic
of

very cumbersome. Toreduce the calculation work, the step


deviation method is used in which
are taken.
step deviations of the values of the variable
series), then d, =x,-A
Let A be any assumed number (usually taken in the middle the
of

are the deviations of the values of x from A.


In case of continuous distribution, hving equal class intervals
of width h (say), the value
of step deviations are
A
*;- (1sisn)
h
where A and h are both arbitrary constants.

x=A + Efu h where N= Ef;


N

=A + Eu
Note. For individual series, the above formula reduces to
n
TENDENCY 2.5
MEASURE OF CENTRAL
:
Example 7. Calculate the A.M. from the following data
10-20 20 -- 30 30 O

Marks :
:
No nf siudents 12 18 27 17

Solution, Calclation ofA.M.:


26
Marks No. of students ) Mid values (*;) u,
10
(A 25: h 10)

0- 10 12
10 - 20 18 15 1 -18
20-- 30 27 25
30 – 40 20
1
20 35
40- 50 17 45 2 34
50 - 60 6 55 3 18

N= 100 £f;u, = 30

Now, A.M. (T)= A+2i4y h =


+10
25
30 =
28
NJ 100
A.M. (E) =28.
Example Weights of trainees in a wrestling coaching camp are given in the table belouw.
8.
Calculate the average weight:
Weight in kg : Below 40- 45 45-50 50-55 55 - 60 60- 65 65-70 70-75 75-80
40
No. of trainees :
5 6 10 12 10 8 4 2

Solution. Here we note that the first class i.e., below 40' is an open class. In the absence
of otherinformation, we assume the first class as 3540. (As the width of intervals in all other
classes is 5,so we also take the width of interval equal to 5 in this case.)
Let the assumed mean i.e., A = 57-5 and h= width of class interval = 5
We may now prepare the following table:

Weight in kg. No. of trainees Mid values u; =


h
() (x) (A = 57-5; h = 5)

35 - 40 3 37-5 -4 -12
40– 45 5 425 -3 - 15
45 – 50 6 475 -2 - 12
50-- 55 10 5215 -1 - 10
55 60- 12 0
60 -65 10 62-5 1 10
65-70 8 67:5 16
70 - 75 72-5 12
75-80 2 77-5
Ef= N= 60 |Eu, -3
123
MATIHEMATICS POR BG.A,
wwHolhyelei

Here A= 57-5, h=5


6x(-3) m 57.5 0-26 e 57.26
A.M.(I)=A+h.57.5+
N 60

the mean weight is 57:25 kg.


IHence
mean marka of all the studenta of
60
data compute the
Example 9. From the following
schools in a city
:
--
16 20 Less thun 16
26 -30 20 26
Marks obtained: More than 35 30-35 rear

: 7 10 15
No. of schools 100
200 160
Average No. of 200 260 300
students in a school mcan
in ascending order and then calculate the
Solution. Let us rewrite the given data
marks. of
are not givon, so first we shall find the total number
IIere total numbcr of students schools with avernge number of students.
of
students by multiplying the number :

Calculation ofmean marks


-A
Averngo no, of Total no.
Marks Mid values No. of = = 5)
students in a school of students (() (A 27-5, h
(x,) schools
-
400 -3 1200
100
J0- 16 12-5
750 -2 - 1500
150
15- 20 17-5
1800 -1 - 1800
20 - 25 22-5 200
--
30 27-5 15 300 4500
25
10 250 2500 1 2500
30- 35 32-5
7 200 1400 2 2800
35 -40 37-5
N= 11350
= 800

800 =
A.M. (E)= A+ xh= 27·5
+
x5 = 27-5 + 0-35 27-85.
N 11350

Hence the man marks is 27-85.


2.7. A.M. of Combined Group

If X, and Xg are the A.M.'s of two groups having n,and n, items, then the A.M. (X) of
the combined group is given by

X =

Note. This formulu can be extended to n number of groups.


MEASURE OF CENTRAL TNDENCY
2.7
2.8. Important Properties of Arithmetlc Mean
1. The agebraic sum of
thedeviations from arithmetic meun is ulways zero
n
E
(,- )=0 or for a frequeney distributionnf(x,
i.e., -) =0.
2. The sum of squares of the deviations of items fron A.M. is least

ie., Zi-z)° is least.

Example 10.The mean wage of 1000workers in a paper millrunning two


400 workers is Rs. 1000. The mean wage of 600workers shifts of600 and
in the Ist shift is Rs. 900. Find the îmean
wage of workers working in the llnd shift.
Solution. Number ofworkers in Ist shift (n,) = 600
Number of workers in IInd shift (n) =400
Mean wage of workers in Ist shift (X, ) =Rs. 900
Mean wage of workersin IInd shift (X,) =?

Mean wage of all workers (X) =Rs. 1000


Now,
[Refer Art. 2.7
n + n2
600 (900) + 400 (X,)
1000 =
600 + 400
Or 1000000 =540000 +
400 X,
Or 400 K, =460000

460000 =
Rs. 1150.
400
Example 11. The averáge weight of 150students in a class is 80 kg. The average weight
boys in the class is 85 kg and that ofgirls is 70 kg. Find the numbers of
ofboys and girls in theclass
separately.
Solution. Let number of boys in a class (N) =*
Number of girls in a class (N,) = 150 –x
Average weight of boys (X, ) = 85 kg
Average weight of girls (X,) = 70 kg
s
Combined average weight of
whole class (X) = 80 kg
N,X, + N,x
Now, using formula
+
N
N,
MATHËMATICS FOR B,CA, -123

x
(86) +
(150*) 70:
80
70%
80 %
150 85x+ 10500
0T
16: 12000-10600
15x 1500

Number of boysin a class 100


=
number of girls in a clasg 150
- 100 50.
And below:
12. From the following results of two colleges A and B given
Example
wlhy ?
State which of them is better and
o College A College B
Name of Exam.
Appeared Passed Appeared Passed
180 108 240 200
B.B.A.
240 192 200 150
B.C.A.
260 t 176
B,Com. 300 250
80 56 100 80
M.C.A.
Total 800 606 800 606

Solution.
Name of Exam. College A College B
Appeared| Passed PassX,W, Appeared | Passed Pass XWB
WA % age o age
B.B.A. 180 108 60 10800 240 200 83-333 20000
B.C.A. 240 192 80 19200 200 150 75 15000
B.Com. 300 250 83-333 25000 260 176 67-69 17600
M.C.A. 80 56 70 5600 100 80 80 8000
Total 800 606 293-33 60600 800 606 306-02 60600

Now, Average pass % of college A = 293-33


n 4

Average pass % of college B = =


306-02
n 4

Since average pass % of college B is greater than average pass % of college A, therefore we
can conclude that college B is better than college A.. However as the
the conclusion is not
average pass % is affected by the number of students appearing in the examination validin different
cOurses. Thus, appropriate average would be weighted average to decide which college is better.
ATASURE OF CENTRAL TENDENCY
2,9

Now, weighted average of college A EX,WA


£ WA

60600 =
75-75
800

and weighted average of college B =

X 60600
=75.75
800
By comparing the weighted means,we conclude that both colleges A and B are equally
good.

2.9. Merits and Demerits of Arithmetic Mean


Merits :
1. It issimple and easy to calculate.
2. It is welldefined.
It is based on all observations and thus is a better representative of the data.
3.
4. It is a good standard for comparison.
5. It is not much affected by fluctuations of sampling.
6. Further algebraic treatment of arithmetic mean is possible.
Demerits :
1. It is unduly affected by extreme values.
2. It cannot be determined accurately unless all the items are known.
3. It cannot be used in the study of ratios, rates etc.
4. Its value may not be present in the given data.
5. It cannot be determined by inspection nor can it be located graphically.

EXERCISE 2.1
1. Calculate the arithmetic mean of marks obtained by 10 students in mathematics test
:
given below
52,40, 70, 52, 43, 40, 65, 35, 48.
75,
2. (¿) Find the average of first n natural numbers.
(i) Find the average of squares of first n natural numbers.
3. Find the average marks obtained by the students of a class from the following data :

Marks obtained: 36 42 46 55 63 72
No. of students: 4 10 4

4. (i) The mean of 10 observations is 25. If each observation is increased by 4, what is the
mean of the new set ?
MATHEMATICS roR B,C.A. 123

each observation,
2 marks are doducted t0
) Tho mean observations is 12.76, If
of 20

show that the mean marks are also


roduced by 2.
WAs ealeulated to be 40 kg.
It was later discovercd that one
boyN mean
5, the RVerAge weight of 18
of correct weight 63. Calculate the correct
weight was misread
as 38 in place
mean fom tho following data
t. Cakulate the arithmetic .
G0 60-0| 70- 80 80 9090T
+460 4050 50-
Class interral j i0- 20 20-S0 30
17 29
trrqueney mean
following data was obtaincd. Find the arithmetic
7 In a study on patients the
3039 40 - 49 50-- 7079
59) 60--69 80 -89
Age inyears): 20 1019 -29
10 17 38 9
No. of cases:
firom the following data
8. ind the avernge marks of the students
10 20 30 40 50 60 70 upto 80
Marks (more thand:
97 87 73 50 25 6 2
No. of students: 100

from the following data


9, Find the average marks of the students

Marks (below): 10 20 30 40 50 60 70 80

No, of students: 15 35 60 84 96 127 198 250

10. The following table gives the weekly wages in rupees of workers. The frequency of the
-
class 49 52 is missing. It is known that the mean of the frequency distribution is 47:2.
Find the missing frequency :

Weekly wages (Rs.): 40- 43 43 – 46 46- 49 49 -52 52-- 55

Number of workers : 31 58 60 27

11. From the following information

Factory A
Factory B
No. of wage earners 250 200
Average daily wages Rs. 2 Rs. 2·50

find out (i) which factory pays larger amount as daily wages ?

what is the average daily wage of the workers of two factories taken together ?
(iü)

12. Fifty students took up a test. The result those who passed the test is as given below:
of

Marks: 4 5 6 7 8
No. of students: 8 10 4 3

If the average of' all the 50 students was 5-16 marks, find the average of those who failed.
TENDENCY
JËASURE OF CENTRAL 2.11

The average monthly wage of a group 10 persons is Rs. 1500. One member of the group.
of
twhose monthly wage is Rs, 1300, 1eft the group and is replaced by a new member whose
honthly wage is Rs. 1200. Find the new average monthly wage.
The mean weight of 150 students in a certain class is 60 kg. The mean weight of boys in
the class is 70 kg and that of girls is 55 kg. Find the number of boys and girls.
18 The mean wage of 2000 workers working in a factory is Rs. 1500. The mean wage of 750
urorkers of the first shift is 1600. Find the mean wage of the rest of the workers.

16 The following information relates to the wages of workers in a factory, their total working
hours and the average working hours for workers, calculate the mean wage per head.

Wages (in Rs.) 100 - 200 200-300|300


-
400 400- 500 500 - 600 600-700
Total hrs worked: 100 150 180 90 70 60
Average No. of hrs
worked per worker) 10 7-5 6 5 7 5

17. Comment on the performance of the students of two universities given below :

Name of University A University B


Examination| No. of students appeared Passed No. of studets appeared Passed
B.A. 600 500 2000 1600
B.Tech. 1000 900 2400 1900
B.B.A. 4000 3000 2000 1500
B.C.A. 2400 1400 1600 1000
Total 8000 5800 8000 6000

18. Comment on the performance of the students of the three colleges given below using
simple and weighted averages :

Name of Pass 'A Pass 'B' Pass


Examination No. of No. of % No. of students
students students (in hundreds)
(in hundreds) (in hundreds)
B.C.A. 71 3 82 2 81 2

B.B.A. 83 4 76 3 76 3-5

B.Tech. 73 5 73 6 74 4-5

M.C.A. 74 2 76 7 58 2

M.B.A. 65 3 65 3 70 7

B.Sc. 66 3 60 7 73 2
MATHEMATICS FOR B.CA, - 123
2.12
ANSWEAS

(n + 1) (2n +1) 3. 52.75


1. 52 2. (ii)

6. 48.4
4. () 29 5. 41 kg
9, 50.4
7. 60-7 8. 39
(i) Rs. 2-22 12. 21
10. 44 11. () Equal 100.
50: No, of girls
Rs. 1490 14. No. of boys
13.
15. Rs. 1440. 16. Rs. 384
Performance of the students of University is better.
B
17.
'A' is best.
18. Performance of the students of college

2.10. Geometric Mean (G.M.) as nth root of the product of all


any statistical data is defined the
The geometric mean of
the n values of the variable.
an individual series of n values of variables *, yy the geometric mean .,
(i) For
G.M. is denoted by G and defined as

we have
Taking logarithms on both sides,
... +
log G= = [log x, + log x, + log x,

n
log x;
= i-1
log G
n

Slog *; 2 log x
G = Antilog i=1 or briefly G= Antilog

:
Geometric..,Mean for Frequency Distribution (Discrete Series) If the variate
(ii)
x has values x,, *,, *, with frequencies f fos ..f,
G= (x'i.xf2. .,
then the G.M. (G) is given by
xn)N, where N = f.
Taking logarithms on both sides, we have

log G = 1
logx +
log *, t ..+ n log *]
Sf
i=1
log *;
log G
MEASURE OF CENTRAL
TENDENCY 2.13

log x}

G = Antilog
N

briefly G =
Antilog 2f log x
or N
2.11. Weighted Geometric Mean
If in any data, the values of the variable are not of equal importance or are of varying
significance, then the weighted G.M. is given'as

n
w, log *

Weighted G.M. = Antilog


n

where , w,, .., w, are the weights of the corresponding values


under consideration.
x, x y., x
of the variable

SOLVED EXAMPLES

Example 1. Calculate the geometric mean of the series 10, 110, 120, 50, 52, 80, 37, 60.
Solution. Calculation of G.M.:

.S.No. log x
1 10 1-0

2 110 2-0414
3 120 2-0792
4 50 1-6990
5 52 1-7160
6 80 1-9031
7 37 1-5682
8 60 1-7782

n=8 2 log x; = 13-7851

(2 log ;
Now G.M. = Antilog

(13-7851
G= Antilog
8
= Antilog 1-723 = 52-861.
MATHEMATICS FOOR
B.C.A.
-123
2.14

:
Example 2. Find the G.M. from the following
30 40 50
10 20
Marks obtained(below): 72 92 100
12 27
No. of students:
Solution. Calculation of G.M.:
Mid value (x) log x f.Jog x
Marks No. of students (f)
0-6990 8-388
6
12 17-6415
0- 10 15
1-1761
27- 12 15
10-20 1-3979 S 62-9055
25
72 -27=45
20-30 35 1-5441 30-882
30 - 40 92- 72 20 1-6532 13-2256
45
50 100 – 92 = 8
40- =
133-0426
Eflog
N= 100

(Ef log x
G.M. = Antilog
Now, N

- Antilog 133-0426 = Antilog (13304) =21-40.


G.M.
100
the
geometric mean of the four items 8, 25, 17 and 30 is 15-3. If
Example 3. The weighted respectively, find the weight of the fourth
item.
weights ofthe first three items are 5, 3 and 4
w,
Solution. Let the weight of the fourth item be :

Caleulation of Geometric Mean


x
log x W;. log

5 0-9031 4-5155

3 13979 4-1937
25
17 4 1-2304 4-9216

30 1.4771 1.4771 W

W1

E w, =
12 + w 2 w; log x
= 13-6308 + 1-4771 w,

Ž w; logx
Now, G.M. = Antilog

Here G.M. - 15-3 (Given)


w,
153 = Antilog 13-6308 +1-4771
12 + W1
Taking log on both sides, we have

log 15-3 =
13-6308 +1-477l w
12+ W;
MEASURE OF CENTRAL TENDENCY 2.15

1-1847(12 + w,) = 13-6308 + 1·4771


w

14-2164+ 1-1847 w, 13-6308 + 1·4771w,


1-1847 w,-1·4771 w, = 13-6308- 14-2164
w,
-0-2924 =-0-5858
0-5856 = 2.003 or 2 (approx.)
W1
0-2924
Jlence, the weight of the fourth item is 2,
Bxample 4. The G.M. of 20items was found to be 10. Later on it was found that one item
78was misread as 8. rind the correct value of the G.M.
Solution. In the given problem,
Number of items = 20
Incorrect G.M.= 10
Correct item = 18
Incorrect item=8
log x)
Now, G= Antilog
x
Incorrect 2 log
10= Antilog|
20
Incorrect E log x
Or log 10 =
20
= 20
Incorrect E logx= 201og 10 = 20 (1-000)
Now, correct log x= 20- log 8 + log 18
= 20- 0-9031 + 1-2553 = 20-3522

= 20-3522 =
Antilog (1-0176) = 10-41.
Correct G.M, Antilog
20
Example
5.

Ifarithmetic mean and geometric mean oftwo values are 10 and 8 respectively,
find the two values.
Solution. Let two values be a and b. Then
a+b = 10 20 ..(1)
a+b
and Vab = 8 ab = 64 ...(2)
= (a + b)2-4ab
Now, (a-b)

Va + b)² 4ab
Or a -b= t
Or a-b=t J(20) - 4 x 64 = tJ400 – 256 = + 12

(Using (1) and (2))


u-b= 12 ...(3) [: a-b=- 12 is rejected]
Solving (1) and (3), we get a= 16 and b=4.
2
MATIENATHS KOR hEA=

I, aN ,
IR QeMnatrie lean o (Comblned Oroujy
re e (AL, of tuo grus Aoving n, und n, itema, then
the G., of the

0
atutian 1,

(lmbine (0.AM, Atilhg


10 log20+ 20 log 18|
- Antilog 10+ 20
10(1-9010) + 20(1.1761)]
Antilog 30
13 01+ 28 52 n Antilog|
= Antilog T8trecenert
30 30
=Antilogs [12177] 16-51,

A13 Averagng ot Peroentages


auerage rate ofincrease or decrease in the
NHetrc mean is generally used to find he
Npulatin, sales,

th
IfV and
perind
, uctin, st pries, ete.
arv the values of a variable at the beginning of the first and at the bnd of
wìvely, andris the average rate of grovth
per unit, then
the

ample 7. fhe rce of the commodity is doubled in 5 years, find the annual rate of
Salution, let a be the value at beginning èe. V, Rs,
r will bee the value in 5 years å.e,, V
Let b the arerage rate of inerease per unit
Rs, 2r

+ r)
Then Vy=V(l

2=(l + ro
Taking log on both sides, we have
S log (l +t r) log 2
5 log (1 +r)a03010
log (1 + )a00602
=
1
1+raAntilog (00602) 1148
r=l14S -1 0:148
= 0: 148 x 100 = 14-8%
Averag perentage rate of increase
MEASURE OF CENTRAL TENDENCY
2.17
Example 8. If the price of a conmmodily increases by 10 h in the first year, decreases by
on in the second year, agan increases by 10 % in the third yeut, Findl veragp
inerease / decrease in the price of the commodity in three years.
Solution. Since we are dealing with % rate of increase or decrease in the price of a
commodity, the appropriate average to be computed is the Geometric Mean.

Year Rate of Increase / Decreasc Price at the end of year (X) log X
+ 10 %
110 2-0414
2 - 20 % 80 1-9031
3 + 10 % 110 2-0414
Total £ log X
= 5-9859

G.M. = Antilog 2 log


X
- Antilog 5.9859
3
= Antilog 1-9953 99-15.
Hence, average rate of decrease = 100- 99-15 = 0-85 %
2.14. Merits and Demerits of Geometric Mean
Merits:
1. Geometric mean (G.M.) is rigidly defined.
2. It is based on each and every observation of the series.
3. It is suitable for further mathematical treatment.
4. It is a proper average to measure the relative change. G.M. is a suitable average to
find the average percentage increase in population, production, sales over a period of
time.
5. Itis useful in average ratios and percentages and in determining rates ofincrease and
decrease.
6. It gives higher weightage to the smaller valucs and smaller weightage to the larger
values than does the arithmetic mean. It is because of the reason that G.M. is never
greater than the A.M.
Demerits :
1. Geometric man is not easy to understand and to calculate.
2. It is difficult to interpret and so has restricted application.
3. If any one or mnore of the observations are zero, then geometric mean cannot be
computed. Aiso, if any one ofthe observations negative,then geometric mean cannót
is
be found because it becomes imaginary regardless of the magnitude of the other items.

EXERCISE 2,2
l. Find the Geometric mean of
the following series:
10 12 T4 16 18
:
6 10 20
130 135-139
- 3
T-9 9-0S 5-

Weigts

Ctting
230
12

laeS

ANSWER

3. R. 111: Rs. ST4 4. 2175


7-2
..3
9.
5
1. 233
= 4g63%
12. Mais =513*: Feales
MEASURE OF CENTRAL TENDENCY
2.19
2.15. Harmonlc Mean
The harmonic mean of any stalistical data is the quotient of the number itema
of divided by
the sum of the reciprocals of all the values of the variable.
(i)For an individual sories of n values of variables *,, X..., *,, the harmonic mean
H.M. is defined as

H.M, =
1.1.l 1

(i) For Frequency Distribution :


If thevariate : has the values x, * ..,.*, with frequencics f, fa., Sp then the H.M. is
defined as

MEM

HM.= +f t..fh

N , EM
where N=

or N
briefly H.M.=

Note. Forgrouped frequency distribution; x,denotes the mid-value ofith class.


Remark. Harmonic mean is useful for computing theaverage rate of increase in profits of a
concern or average speed at which a journey has been performed or the average
price at which an article has been sold.
2.16. Weighted Harmonic Mean

If in any data, the values of the variable are not of equal importance or are of varying
significance, then the weighted HM. is given by

W;

Weighted H.M,=

where w,, wa., w, are the weights of the corresponding values of


x, *y of the variable
under consideration.
124
MATHEMATICS FOR B.C.A.
2.20
SOLVED EXAMPLES

series;
mean (,M.) for the following individual
Example 1. Calculate the utnonie 19.
2, 4, 7, 12,
Solution, Calculation of larmonic mean (HM.)
1

S.No.
0-5000
2
0-2500
4
0-1429
7
0-0833
12
4 0-526
5
19
1-0288
N=5
N 5
=
=
4-86.
lHarmonic mean (H.M.)
) 1-0288

of 25) obtained by a
group of students in
table gives marks (out
Example 2. The followingmean
a test. Calculate the harmonic of the series.
22 23 24 25
: 20 21
Marks obtained
: 4 7
No. of students
mean (H.M.):
Solution, Calculation ofHarmonic
1 No. of students (f) f.
Marks (x)
4
0-20000
20 0-05000
2 0-09524
21 0-04762
7 0-31815
22 0-04545
1 0-04348
23 0-04348 H

0-12501
24 0-04167
1 0-04000
25 0-04000 H

= 0-82188
N= 18 £f.-
N 18 = 21-9.
Harmonic mean (H.M.)= 0-82188

Example 3. Calculute the H.M. for the following:


Murks ) 10 10 - 20 20 -3 30 - 40 40- 50
No, of students: 7 28 12
MEASURE OF CENTRAL
TENDENCY
2.21
:

Solution. Calcilation
of
i.M.

Marks No, of students () Mid valuo (¢) flx


0
10 0-8000
10 -20 7 16 0:4667
20 - 30 28 26 1-1200
30 - 40 12 36 0-3429
40-50 9 46 0-2000

N= 60 = 2-9296

N 60 =
H.M, = 20.48.
E(4) 2-9296

Example 4. The Harmonic Mean (H.M.) of 100 observations is found to be 17. Later on it
was discovered that one item 20 was misread as 28. Find the correct value of the H.M.
Solution. Here, number of items = 100
Incorrect H.M. = 17
Correct item = 20
Incorrect itemn= 28
N
Now, H.M, =

100
Or 17 =
1
incorrect

Incorrect S 100
17
S 100 1 1
Corroct =5-8966
17 28 20
100 100 =
Correct II.M, 16-9589.
5-8966

2.17. Harmonic Mean of Combined Group


If H, and H, are the II.M. of two groups having N, and Ng items, then the H.M. of the
combined group is given by
H.M, =
N, +N
N
H
N,
H,

Note. This formula can be extended to n number ofgroups.


B.CA. - t2
MATHEMATICS FOR
2.22
30 items is 40. Find the
h.M. of
items is 25 and that of
Dxample 5. The H.M. of 20
combined group.
Solution. Here N, = 20, N, =30
=
H, = 25, H, 40
group.
Let H be the H.M. of combined
N, + N 20 +30
=
H N, 20 30
Then N 25 40
H, H,
50 50 32-268.
1-55
0-8 +0.75
of speeds.
average of speeds. For averaging
2.18. Averaging of Speeds finding the
Harmonic Mean is
generally used in
Simple H.M. is used, if covered at different speeds (x),
distances are (x) are maintained.
() equal periods, different specds
() for equal timcused, if corresponding distances
Weighted H.M. is speeds (x). Here
distances are covered at different
{i) unequal corresponding times
(w).
are taken as weights speeds are maintained.
Here
periods, different
iü) for unequal time the first
are taken as weights (w). car days. He covered 480 km each day. On 40 km
for 3 at
Example 6. A man
travelled by on the second day he droue for 12 hours ?
drove for 10hours at 48 km an hour, km. What was his average speed
day he
an hour and on last day he
drove for 15 hours at 32
480 km
travelled on each day is constant, i.e.,
Solution, Here the distance H.M.
.. Average speed can be calculated by using simple
= N
H.M. 1
1
Now, 1

3x480 :38-92 kmh


1 37 37
1 1

48 40 32 480 km
covers 3 km at an average speed of& km per hour, unother 2
Example 7.A cyclist first entire
km at 2 km per hour. Find the average speed for the
at 3 km per hour and the last 2
journey.
Solution, W

Speed («) Distance (w)


(km hr) (in km)
0-375
8
0-667
2 2 1-000
2w = 7 E wlx = 2-042
MEASURE OF CENTRAL
TENDENCY
2,23

7
= =3•429 km/hr.
Average speed
2.042

Example 8. The rate of a certain commodity in the first weeh of January2009 is O-4 hg per
per rupee in the third weeh. Examine
D2e is 0-6 kg per rupee in the second week and 0:5 kg
rupee ?
hether is it correct to say that the average price is 0:5 kg per
Solution. Here the given average price is 0-5, which is based on arithmetic mean. But in
this case appropriate average is harmonic mean and not arithmetic
mean, The correct angwer
:

isthe H.M. which is given below


H.M. = N
1
1,1,1 1, 1
b 0•4 0-6 0-5
0-3+0-2+0-24 3
12 0-74
0-12
x0-12 =
3
0-486.
0.74
Thus the given average price viz., 0-5 is not correct.
IHonce the average price is 0-486 kg per rupee and not 0-5 kg per rupee.

2.19. Merits and Demerlts of Harmonic mean

Merits.
1. Harmonic mean (H.M.) is rigidly defined.
2. It is based on all the observations.
3. It is suitable for further mathematical treatment.
4. Since the reciprocals of the values of thevariable are involved, it gives greater weightage
tosmaller values and as such is not very much affected by one or two large values.
5. Itis particularly useful in averaging special types of rates and ratios where time factor
is involved.
Demerits.
1. It is not easy to understand and to calculate.
2. Its valuecannot be computed if there are both positive and negative observations in a
series or if any one or more of the observations are zero.
3. It is not a representative figure of the distribution unless the phenomenon requires
greater weightage to be given to smaller items. Because of this limitation the harmonie
mean is not a good representation of a statistical series.
2.20. Relation Between Mathematical Averages
In any distribution, A.M., G.M. and H.M. differ in their values. Generally they differ in the
following order:
A.M. > G.M. > H.M.
Also, (G.M. =(A.M.) (HM.)
- 123
MATHEMATICS FOR B.C.A,
MEA
2.24
positivenumbers, then
Theorem. Ifx andy are two 3.
> H.M.
() A.I, > GM,
() (G.M4.) (A..) (H.M.) positive numbers and x +y.
=
y two
Proof. () Let and be
x
2xy
G. M, = Jy; H.M. 4
A.M. *+);
2
G.M.
Now A.M. will be greater than
A.M, -G.M. >0
if
Jy >0
i.e.,, if 2

x+y- 2 /xy >0


ie., if
(V-) >0
i.e.,if (Square of any real quantity is +vel
...(1)
which is true.
A.M, > G.M.
H.M., if
Now G.M. will be greater than
2xy
L

i.e., if
*+y >2/xy

i.e., if
*+y-2Jxy >0
Wa-y >0
i.e., if (Square quantityl
which is true. ..2)
G.M. > H.M.

From (1) and (2), we have A.M. > G.M. > HM.

(iü) Now (A.M.) (H.M.)

= 1y = (G.M.2
(G.M. = (A.M.) (H.M.)
EXERCISE 2.3

. Find the harmonic mean of the individual series :


3, 5, 6, 6, 7, 10,

2. The following table gives the marks (out


of BO) obtained by students in a class. Caleuke
the H.M.

Marks: 18 21 24
30 38 45
26
No. of students: 6 12 15 19 9 2
MEASURE OF CENTRAL TENDENCY 2.25
:

3, Find the 11.M. of the following distributon

24 8 - 10
Class:
20 0 30 10
Frequency :

4. Calculate the H.M. for the following distribution

Income(in Rs): 10 20 30 40 60
No. of persons : 2 4 3

5. Tho H.M. of 150 items is 17 and of 250 itoms is 22. Find tho H.M, of tho combinod group.
6. The lH.M. of the 200 observations is found to be 18. Lator on it wns discovorod that ono
item 20 was misread as 18, Find the corroct value of H.M.
7. The H.M. of 50 items was found to be 126. Later on it was discovorod that two itema 100
and 200 were misread as 90 and 250 rospectively. Caleulato the corroct valuo of tho H.M.
8. A motorist covers distances 20 km, 30 km, 10 km at the spoods of B0 lkm/hr, 60 km/hr and
22 km/hr respectively. Calculate the averago speod of the motorist.
9. A car covers lst 30 km at an average speed of 80 km/hr, another 20 km at 30 km/hr and
the last 20 km at 20 km/hr. Find the average spoed for the ontire journey.
10. A man travels 50 miles at a speed of 20 miles per hour and thon roturns at a spood of 30
miles per hour. What is the average speed for the wholo journoy ?
11. Mr. Verma of Sonepat started for village which was at a distance of 6 km. He travelled in
his car at a speed of 40 km per hour. After travelling for 4 km the car stoppod running.
Then he traveled in a rickshaw at a spoed of l0 km per hour. Aftor travolling a distance of

1 km he left rickshaw and covered the romaining distance on foot at a spoed of 4 km per

hour. Find the average speed per hour of Mr.Verma.

ANSWERS

1. 5-9 2, 25-09 3. 4-98

4. Rs. 19.23 5. 19-8146 6. 18-0090


7. 125-0347. 8. 44-2968 km/hr 9. 34-3 km/hr
10. 24 miles per hour 11, 16 km/hr
2.21. Median
Median is that value of the variate which divides the total frequency into two equal parts,
when the variates are arranged in ascending or desconding order of their magnitudo.
2.22, Calculation of Medlan

a) For an individual serics. (Ungrouped data): First arrango the data in asconding or
descending order ofmagnitudes. Let n be the numbor of itoms.
123
MATHEMATICS FOR B.C.A.
2.2%

case the value of item is the median.


Case I, When n is odd. In this

even, In this case the mean of the values of |th and


II, When n is
Case
items is the median. :
(6) Fora frequency distribution arrange the data in ascending or descending
we
()) When the series is discrete Here first
:

magnitudes and then prepare the cumulative frequency table. Let N be the total
order of
frequency.
Median is the value of th item
:

(i¿) When the series is continuous


case of grouped continuous frequency distribution, first of all we find the median class
In
by the formula
(the class in which th item lies is called median class). The median is given
2
N
2
Median=l+
f
lower limit of median class
where l= -
h = width of the median class (i.e., upper limit lower limit)
f= frequency of the medianof class.
class.
C=cumulative frequency the class preceeding the median
to an exclusive form.
Note. To find the median in case of distribution of inclusiveform, first convert it
SOLVED EXAMPLES
:
Example 1. Find the median for the series
10, 12, 8, 9, 70, 60, 40, 80.
Solution, Let us first arrange the values in ascending order which is
8, 9, 10, 12, 40, 60, 70, 80
Here n =8 i.e., an even number
n
th item + +1|th item
Median =
2
4th item + 5th item 12 + 40 = 26.
2 2
Example 2. Find the value of median for the following series :

4, 6, 7, 8, 12, 10, 13, 14, 3.


Solution. Let us first arrange the values in ascending order which is
3, 4, 6,7, 8, 10, 12,13,14 ...1)
MEASURE OF CENTRAL TENDENCY 2.27

Here n, the number of items is 9 i.e., odd


item
Median=
-eh iitem 5th item

the series (1) is8


Now, 5th item in
Median=8.
Example 3. The median of the following observations, arranged in ascending order is 42;
find x:
22, 24, 33, 37, x+1,
x+3, 44, 47, 51, 58.
Solution. Here the number of items is 10 i.e., even
(10
Median = Average of
2 -+1th
th and items

= Average of 5th and 6th items


+
(x+1) (x +3) 2x +4 =x +
2
2 2
Nowthemedian is given tobe 42. Thereforex +2= 42 x = 40.
Example 4. The following is the distribution ofwages of200 employees in a factory. Calculate
the median wage :

Daily wages (in Rs.): 210 160 200 150 170 180 230 220 140 190
No. of employees : 15 20 18 35 27 23 13 12 18 19

prepare the cumulative


Solution. First we arange the data in ascending order and
:
frequency table
Daily wages (x) No. of employees () Cumulative frequency (c.f.)
140 18 18

150 35 53

160 20 73

170 27 100
180 23 123
190 19 142
200 18 160

210 15 175
220 12 187
230 13 200
N= 200
Here N= 200, which is even
MATHEMATICS FOR B.C.A,
.
2.28

(200)
and
200 +1 th items
Median Average of

2
th 2
= Average of (100)th and (101)th items 101th item has
100th item has value 170 and
Now, from above table, we observe that
value 180
170 + 180 350 = 175
Median =
2

Median Rs. 175. = :


distribution
median for the following -
Example 5. Find the
|28-35 35 -42 42 49
Marks: 0-7 7-14 14-2121-28 16
7 11
3 4
No. of students:
:
of Median
Solution, Calculation
Cumulative frequency (c.f.)
No. of students (f)
Marks (x)
3
0-7 7
4
7- 14 14
7
14- 21
25
11
21-28 25
28 - 35 41
16
35 – 42
50
42 - 49
N= 50
Here N = 50,
N 50
Median= Size of th item = Size of 2 =25th item
2
Median class is 21- 28
Here l= 21, h=7, f= 11, C= 14
Median= l+ -)
7
= 21 + (25- 14)
11

= 21 + 7x11 = 23.

Example . 11
The following table gives the marks obtained by 50 students in mathematics
Find the median.

Marhs: 10 14 16 -,19 20 -24| 25 - 29 | 30 - 34 35- 39 40 -


44|45 -49
No. of
students: 5 8 6 7 6 6
JEASURE OF CENTRAL TENDENCY 2.29

Solution. Let us first prepare ah exclusive table which gives the frequencies and cumulative
reguencies. (Here the gap between upper limit of a class and lower limit of next class is 1.
1
Hence subtract from lower limit and add 05 to upper limit of each class to convert the
= 0-5
2
inclusive class interval into exclusive class interval)

Marks Frequency Cumulative frequency (c.f)


9-5 - 14-5 5
14-5 - 19.5 13

195 -24-5 i9
24-5 - 29-5 7 26
29-5 - 34-5 6 32
34-5 -39-5 3 35
39 5 - 44-5 44
44-5 – 49-5 6 50
N = 50

Hre N= 50,

N
Median = Size of th item
2

= Size of 50
th = 25th item

Now, the cumulative frequency 25 belongs to the class 24-5 - 29-5


Median classis 24-5 – 29-5

h= 29-5-24-5 = 5, f=7,
=
Here 24-5, C= 19

Median = l+:

= 24-5 + 5 (25 -19)


7

= 24-5 + 5x6
7
= 24-50 + 4-28 = 28-78.
Bxample7. From the following wage distribution, find out the value of median.

Monthly wages:
50-80 80- 100 100- 110 110- 120| 120-130 130 - 150 150-180 180 - 200
No. of worhers :
30 127 140 240 176 135 20 3
MATHEMATICS FOR B.C.A,
- 122

2.30

Solution, Calculation of Median:


Cumulative frequency (c.f.)
Monthly wages No. of workers (f)
30
50 -80 30
157
80 - 100 127
297
100 – 110 140
537
110 – 120 240
713
120 - 130 176
848
130 – 150 135
868
150 - 180 20
871
180 - 200
N= 871

Median= Size of -th item


2
871 = 435-5th item
= Size of
2

Median class is 110- 120


240, C= 297
Here l=110, h= 10, f=
Median =
+H
= 110 +
10
(435-5 - 297)
240

= 110 + 10 (138-5)
240

= 110 + 138-5
24
= 110 + 5-772 = 115-722.
Example 8. Find the missing frequencies in the following distribution ifN= 74 and median
of the distribution is 36.

Marks : 0-10| 10- 20 20-30 30 - 40 40- 50 50-60 60-7070- 80

No. of students :
2 20 12 4 3

Solution. Here Median =36

and N= 2f= 74
Let a be the freguency of class 20 -30 andb be the frequency of class 50 -60.
20- 30
30- 40 20
40 - 50 12 42
50 - 60 42
B0- 70
70- 80
= 36
Median of the distribution {Given)
Median class is 30- 40
| 30, h 10, f 20, C- 104 a

Median
h (N
=l4

36 = 30 4
10
(37 - 10 - a)
20

6 27-a)

=
12 27-a a= 27- 12 = 15

Also, 49 + a
+674 a +b= 25

b= 25 - 15 = 10
.. Frequency for class 20 -30 is 15

and frequency for class 50 - 60 is 10.


2.23. Graphical Location of Median
Median can also be determined graphically by constructing two ogives-one less than ogive
and other more than ogive. From apoint at which these two ogives intersect, a perpendicular is
drawn on a-axis. The value thus obtained on the x-axis is called the median,

Example 9. Find the median from the fullowing data using ogives :

Wages tin Rs): 20 - 40 40 - - - 160


6060- 80 80- 100 200- 120 220 14o40
No. of uworkers :
3
- 121
MATHEMATICS FOR B.C.A.
2.32

Solution, Caleulation of nedian:


For mnore than ogive
No. of For less than ogive No. of
Wages
Wages No. of workers
IWages workers more than
less tlhan workers
20 58
20
20 - 40 40
40 54
10
10 - 60 6 60 48
20 60
80
60 - SO 10 80 38
100 36
S0 - 100 16
100 22
120 48
100 – 120 12
120 10
140 55
7 3
120- 140 53 140
160
140 – 160 3 160

YA
Less than
60 ogive

Si 50+
40

J0 30
ON More than
20 ogive

10+

20 40 60 80100 120 140 160

Median=91-25

Fig. 2.:1

From the graph, Median =91-25.


2.24. Merits and Demerits of Median
Merits:
1. Itis rigidly defined.
2. It is easy to understand and calculate.
3. It is not affected by the extreme values.
4. It can be located graphically.
5. IL is best suited for open-end classes. Since only the position and not the values ofitens
nust be known. The median is also recommended if the distribution has unequal classes,
sinCe it is easier to compute than the mean.
6. It is Useful when the data cannot be masured quantitatively.
4EASURE OF CNTRAL TENDENCY 233
:
Demerits
1, It is not based on all the items in the given data.
9. It may not be true representative of a given data in many cases.
3. It is not suitable for further algebraic treatment.
A For finding out the median of a data, it is necessary to arrange the
data in ascending
or descending order. In case the number of
items is very large, this process becomes
tedious.
5. The value of median is affected more by sampling fluctuations than the value of the
arithmetic mean.
EXERCISE 2.4
1 :
Bind the median for the following series
25. 20, 23, 32, 40, 27, 30, 25, 20, 10, 55, 41
9 :
Find the median from the following data

5 10 15 20 25
2 4
f: 6 10

:
8. The following table gives the weekly expenditure of 100 families. Find the median

Weekly expenditure
0-10 10- 20 20-30 30-40 40-50
(in Rs.):
No. of families 14 23 27 21 15

:
4. Find the median for the following frequency distribution

Marks: 0-10 10 – 20 20-30 30- 40 40 -50 50 -60


No. of students: 15 17 19 27 19 12

5. The following table gives the weight of 50 students of a class :

Weight (in -
kg): 37-41 42 46 47-51 52-56 57-61 62-66 67-71
No. of students: 3 7 11 14 7 6 2

:
6. Find the median for the following data

130 - 139|140–

Class intervals: 110-119120 -129 149 150- 159|160 169 170-179
Frequency : 25 40 60 40 25 5

7. The following table gives the marks obtained by 80 students in a test. Find the median :

Marks: Below 10 Below 20 Below 30 Below 40 Below 50 Below 60


No. of
students: 3 12 27 57 75 80
4)

3
)
Aiart*

ANSWERS

, 11645
3i, 24.
4. 40
.2%. Mode

2.K0. Aetiods to Eplinete Mode of a Dietritaution

Metbod uf lnnption s du
tha netbud. we širst arrange th ata ig ths fasru uf 4

scry

etiesiyibeeie
rtASURE OF CENTRAL. TENDENCY 2.3:
bamaximum frequency is 9, corresponding to which the value of varíate is 8. Therefore 8 is
the mode of the mven distribution,
Thus, we notice that in discreie series, mode is determined by inspection and therefore,
on error ofjudgement is poSsihle.
Incaseswhere the difference between the maximum frequency
and the fiequency preceeding or succeeding it is verysmalland the items are heavily concentrated
on oither side or in cases when maximum frequency
is repcated, mode can not be found by
inspection. Under such situations the value of mode is detormined by Method of Grouping
which is exxplained in the following example.
Example 1. Find the mode of
the distribution:
5 10 16 20 25 30 35 40
f: 4 15 25 20 17 26 10 3

Solution. The given distribution shows that mode should be 30, corresponding to the
maximum frequency 26. But concentration of valucs also appears near 20 or 15. Therefore, we
:
apply the method of grouping which is being explained below
Method of Grouping
Given frequency 1°requency
III IV VI

5 4

10 15
44
15 25
60
20 20
62
25 17
63
30 26
53
35 10
39
40 3

Method toobtain the above table :

Column I : Write the original frequencies.


Column II : Sum the frequencies taken two at a time starting from lst frequency of
column I.
Column II: Sum the frequencies taken two at a timc starting from 2nd frequency of
column I.
Column IV: Sum the frequencies taken three at a time starting from lst frequency of
column I.
fraqueney of
nt a time starting rom 2ned
(olunn1 Sum the frequeneies tnken three
rnlumn l.
rd frerqtenry f
Sum the frequencies taken three
at A
tma starting frot
Column VI:

Analysis Table
frequeney
items havring maximum
Sire of
Column

20
15
I6
10
25
20 20
15
10
25
20
maximum
items 15 and 20 occurs
the
nbove analysis table, we observe that
From the
4 times each.
number of times i.c.,
frequencies of 15
Sum of censecutive =
15 + 25 + 20 60

Sum of consecutive frequencies of 20


=
25 + 20+ 17 62

Now
62> 60
Mode = 20 Thus
Hence may not be the mode.
maximum frequency
example, it is clear that the variate having
Note. From this
preceeding and succeeding frequencies ofthe variate.
mode is effected by

226.2 For Continuous


Frequency Distribution
a continuous frequency distribution is
of
given by the following formula:
Mode

=
Mode l+hx 2fm-f -fa

of modal class
where l= lower limit
h = width of the modal class
=frequency of the modal class.
class.
frequency of class preceeding the modal
=
f
= frequency of class succeeding the modal class.
f However, if the frequencies are
Modal class corresponds to the maximum frequency.
modalcass is determined by the methou
irregular or maximum frequency is repeated, then the
af grouping.
- TENDENCY 2.37
MATHEMATICS FOR B.C.A, 1)2 MEASURE OF CENTRAL

Of Exampl<2.)Caleulate the mode for the following frequency distribution :


a time starting from 2nd frequency
.
Class interval: 5- 15
16- 26 25-36 36 --15 45 55 G5 .-65
a time starting from 3rd frequency f Frequency : 6 10
Solution. From the given distribution, it is clear that the maximum frequency is 10 and
# ies in the class 25- 35. Thus the modal class is 26-35.
Here = 25, fm = 10, =
f 6,
= 5,
h= 10 ,
ing maximum frequency =
Mode l+hx 2fm
-fi -2
20 = 25 + 10 x -6
10 = 25 10 x 4 =
--
+ 29.44.
15 20 6.-5 9
25
30 Hence, mode = 29-44.
20 Exampld3) Calculate the value of mode for the following frequency distribution :
15
-
20 25 Class : 1-46-89-1213 16| 17
-2021-24 |26- 28 29-3233-36|37-10
5 8 15 11 13
Frequency : 2 9 12 14 14
items 15 and 20 occurs maximum
Solution. Here the classes are not in the inclusive form. so we first convert them in
inclusive form.
:
The given frequency distribution in inclusive form is

– - - –
36-5
Class
: 0-5 -4-5 8-5 12-5 16-5-20-5–24-5 –28-5 –32-5
4-5 8-5 12-5 16-5 20-5 24-5 28-5 32-5 36-5 40-5
: 2 5 12 14 14 15 11 13
Frequency
To find modal class, we shall make use of the grouping method
:

Class Given frequency Frcquency


m frequency may not be the mode. Thus II III IV VI
ievariate. 0-5-4-5 2

:
4-5 -8-5 15
y the following formula
8-5- 12-5 22

12-5 - 16-5 29

16-5 - 20-5 12 35
20-5 - 24-5 14 40
43
24-5- 28-5 14
l class.
28-5- 32:5 15 40
lclass.
However, if the frequencies are ot32-5 - 36-5 11 39
class is determined by the method
36-5 - 40-5 13
MATHEMATICS FOR B.C.A, - 123

Analysis Table

-
Coumn 4- S3- 123-16-5-20-5 245 28-5 32-5 -36-5
20-5 24-5 28-5 325 36-5 40-5
12-3 163

5 4 1 1
Totsl
. Moda dass is 24-5-25-3 I: 245- 28-5 has maximum number of
bars]

Now, Mode = +hx


2fm -f -f2
Here i= 4-5, À = 4, f = 14,
i = l4,
f, = 15

14-14 = 24-5 = 24-5.


Mode = 24-5 + - 15-x4 +0
28-14
2.26.3. Empirical Relationship Between Mean, Median and Mode
Bimodal Series: The distributionwhich has two equal maximum frequencies is knoun
GS bimodal series. (See fig, 2.2)

The process of calculating mode by above formula as discussed in Art. 2.26.2 fails in the
case of bimodal series.

X
Mo Mo

Fig. 2.2
In the case of symmetrical distribution, mean, median and mode coincide.
moderately asymmetrical distribution mean, median and mode are related as
But for

Mean - Mode =3 (Mean- Median)

Or Mode =3 Median -2 Mean


which is the empirical relationship between mean, median and mode.
When the series is bimodal,
the value of mode can be determined by this empirical relationship.
AIEASURE CENTRAL TENDENCY
2.19

Bxample 4. Calculate Median and Mode ofthe data


gien beloo, Using them find arithmetie
mean.
Marks :
10 30
: ssta
No. of st
udents 45
arb

Solution. Caleulation of
Median, Mode.e and Arithmetic Mean: ts

Marks Number of students () Cunulntivo Prequoncy (o)


0- 10

10- 20
23
20 - 30 22
30 - 40 20

40- 50 10 76
50 - 60 5 80
N= 80
Median: Here n = 80 i.e., even

Median = Size of N th item = 80 th item = 40th item


2 2
Median lies in the class 20- 30.
Here l = 20, h = 10, f= 22, C- 23
Median =
+-c
f\2
= 20 + 10 (40 - 23)
22

= 20 + 10 x 17 =20+ 7-73= 27-73.


22
Mode From the given distribution, it is clear that the maximum frequency is 22 and it
:

hes in the class 20-30. Thus the modal class is 20 - 30.

Here = 20, f =
n
22, f, 15, f, = 20, h = 10

Mode = l+hx
fm -fi
2fm -fi-f
-15. = 20 + 10 x– = 20 + 7-78 = 27.78.
=
22
20 + 10 x
44 - 15- 20 9

Arithemetic Mean: We know that, Mode =3 Median -2 Mean


L.e., 27-78 = 3 x 27-73–2 Mean
0 2 Mean=83-19-27-78

55-41 = 27-705.
0r Mean =
2
MATHEMATICS FOR B.C.
2.40

2.27. Graphlca! Locatlon of Mode are drawn diagonaly


of histogram. Two lincs
Mode can also be Ostimated with the help
upper cornors modal class bar to the upper corners of the adjacent ha
trom the of the drawn on x-axis from the point wher
touching the modal class bar. perpendicular
A
is
perpendicular meet the x-axis gives the value of t
these lincs meet. Thepoint whoro this
mode. rog,.
the following series graphically and verify the
Example 5. Calculate the mode of
algebraically.
- 40 40 - 50
10 - 20 20 -30 30
Class : 0-10 21 15
: 14 23 27
No. of students
:
Solution, Graphically

30 2
23 21
20 15
14
10

10 20 30 40 50
0 = 24
Mode
Fig. 2.3
Thus the Mode is 24
Mode = + -xh
Algebraically: 2fm -fi -fa
= 27, = 23, fo= 21, h= 10
Here l=20, fn f
27- 23 x 10
Mode = 20 + --
21
2x 27 -23
= 20 + 4 x 10 = 20 + 4 = 24.
10

2.28. Merits and Demerits of Mode


Merits.
1. It is easy to understand and easy to calculate.
inspection
2. In some cases mode can be located merely by
3. The value of mnode can be obtained graphically with
the help of histogram.
4. Mode is not suitable for further mathematical treatment.
open end classes.
5. It can be calculated for frequency distributions with the
6. Like median, the mode is not unduly affected
by extreme values. Even if frequer
we choose the most
value are very high and the low values are very low
value of the data to the modal value.
KACASURE OF CENTRAL TENDENCY 2.41

Demerits.
1,It is not rigidly defined.
2. Mode is not based on all the values of the data.
3. It isnot suitable for further mathematical treatment.
A. As comparcd with mean, mode is
affected to a greater extent by the fluctuations of
sampling.

EXERCISE 2.5
1.Find the mode for the following series :
3, 5, 6, 2, 5, 4, 5, 9, 5.
:
2. Calculate the mode from the following frequency distribution

:
Size (x) 4 5 6 9 10 11 12 13
Frequency (f): 2 12 14 14 15 11 13

3. Find the mean, median and mode of the following distribution :

Marks 10- 25 25-40 40- 55 55-70 70-85 85-100


Frequency :
6 20 44 26 1

4. The following table gives the weight of 50 students of a class. Find the modal weight.

Weight (in kg): 37-41 42-46 47-51 52- 56 57-61 62-66 67-71
No. of students: 3 7 11 14 7 6 2

5. The marks obtained by 60 students in a class are as given. Calculate the arithmetic average,
median and mode.
Marks No. of students
More than 70% 7

More than 60o 18


More than 50% 40
More than 40% 40
More than 30% 55
More than 20% 60
:
6. Calculate the median and mode of the following

Class : 10- 20 10-30 10-40 10-50 10 -60 10-70 10 -80 10-90


Frequency : 4
16 56 97 124 137 146 150

7. The following table shows the distribution of 100 families according to their expenditure
per week. Number of families corresponding to groups Rs.
10- 20 and Rs. 30 40 are -
;
missing the mode is given to be 24. Calculate the missing frequency.

,•,
l21
MATHEMATÍCS FOR B,C.A,

10 - 20 20 -30 30 - 40 40- 60
nditure: 0-10
16
14 27
No, of families:
ulgebraie method.
following data graphically and veriCy with
nd the mode of the
S 50- 60 60-70
20--30 30 – 40 10-- 50
10- 20 8
B0 12
10 15
ANSWERS

2. 10
1. 5
= 48-57 4. 53-0
Mean = 47-95; Median = 48.18: Mode
3. = 56-67
average = 51-67, Median = 54-54. Mode
5. Arithmetic 8. 34-5.
= 40.67 7. 23; 21
6. Median 44-63, Mode
=

2.29. Partition Values a


the variates wvhich divide the total frequency into
Partition values are those values of
number of equal parts. parts (when
as median divides the total frequency into two equal
:
(i)Quartiles Just magnitude), similarly quartiles divide the total
or order of
arranged in ascending descending of the variate midway between the first variate and
frequency into four equal parts. The value of the variate
as or lower quartile and is denoted by Q,. The value
the median is known First is called the Third or upper quartile
and is
midway between last variate and the median
Q,. The median is known as second quartile and is denoted by Qo.
denoted by In the
of finding the values of Q, and Q, are similar to those of the median.
The methods or descending order of magnitude, Q,, Q,
ase of ungrouped data when arranged in ascending
can be obtaincd by the following formulae:
n +1 Q,= th variate.
th variate;
4
are calculated by the following formulae:
For acontinuous frequency distribution, Q, and Q,
Q
=l+ -)
wherc l= lower limit the class in which a particular quartile lies.
of
h = width of the class in which a particular quartile lies.
f= frequency of the class in which a particular quartile lies.
C cumulative frequency of the class preceeding the class in which particular
=

quartile lies.
(ii) Deciles: These are the values which divide the total frequency into ten equal parts and
are denoted by D, (i = 1, 2,
..., 9).
(iü) Percentiles : These are the values which divide the total frequency into 100 equal
parts and aredenoted by P;(i = 1, 2, 99). ...,
AMEASUJRE ON CENTRAL. TENDENCy
243
9.29.1. Calculationg of Decilesmnd Porcentiles.
(a) Wor Individunl SerioB.

1), the value of


i. th item (where i 1,2,
..., 9)
10

P, the value of i.
In + 1

th item (where i = 1, 2, .., 99)


100
where denotes the totnl numbor of items.
(b) For Discrete Sorios: We u8e the same formula as above. Only n is replaced by N Uhe
total frequency.
(c) For Grouped Frequoncy Distribution :
D, = h(iN -o).
l+ (i= 1, 2, 3,., 9)
f\10
P, = l+ h(iN (i= 1, 2, 3,.., 99)
T100
where l, h,f, C have usual meanings.
SOLVED EXAMPLES

Example 1. Culeulate Q,, y D,, P Poo from the following datu :

Marks
:
0-10 10-20 20 -40 40 -6O 60- 8O 80 - 100
: 5
No. of students 10 22 25 10

Solution.
Marks No. of students () Cumulative Frequency

0 - 10
10-20 10 18
-- 22 40
20 40
40 – 6O 25 65

60- 80 10 75

80 - 100 5 80

N= 80
N 80
Now, th item = th item = 20th item
4 4
Q; lies in the group 20-40
l= 20, h= 20, f= 22, C = 18

Q, = l+
MATHÉMATIC3 FOR B,CA
2.44

4
20
(20- 18) 20 + 1-8 21-8
20
22
3N th item itom
60th
group 40- 60,
Q, lies in the
25, C= 40
l=40, h= 20, f= 20 x 20
Q = 40 +
20
(60 - 40) = 40 +
26
25
40+ 16 = 56,

2 x 80
th item - 16 th item
Also, D, = th item=
10

D, lies in the group 10- 20.


l= 10, h= 10, f= 10, C=8
N
h(2x

= 10 +
10(2x80 -8 = 10 +
8= 18 marks.
10 10

= 5xN th item = 5x 80 th item 4th item


P,
100 100

P, lies in the group 0- 10. C 0


l=0, h= 10, f= 8,
h(5xN
P, =
l7100
10(5 x 80 -o]=5 marks.
=
0+
100
90 xN) 90 x 80
Now, P'go = th item= th item
100 100
= 72th item.

Pgo lies in the group 60 -80.


= 10,C =
l= 60, h= 20, f 65

x N
l+- (90
P.90 = h
100

= 60 + 20 (90 x 80 - 65 = 60 + 2 x
7=74 marks.
10 100
2,45

9.90, Paillon Valuea


Graphlonl ooatlon of
partition valwa l.e,, ndQ,om atolu found HVapbleally ns lollows,
Wo fivst drnw
"e
Lu than yve ol thon lontethe polnt NM on thoyxn, Prom thi
te givenlistilhution nl
iat nplollo thoN Homdrawn, Now on hen poinl where this prnllol lino outathe
jtiealar in drnwn N-xis. "h valuu wlhil this peorponienlar moota the
aia sivn thevaln of
itetlen nul enlen nn Da dlotorndnod with the holp ofogivo
n
imlrly, thn valunol 4, ndan be lond, In the same nanner
of
ixnplo 2, Drwaenn than ogive ofthe fillowng dislrlbution and then flnd the valuea
qatlenyphelly,
- 70-75 -
i0 5 (l6-70 75-A0 80 A5
1)

Bolutio,
66 65 70 75
No, of workors 84 93 111=N

Wi iral (ODALrueta Jeas thun ogivo,

|.066 lhAn
1P0 00lvo
T00

60

10

- 74-77

A20 ofl 27-751)h itom

=04-70 (Qrnphicnlly).

Now, ze of th Item

=4-77raphienlly).
MATHIEMATICS FOR B.C.A. - 1)n
2.46
EXERCISE 2.6
:

1, Find
Q
Qa, 1D,, Pso for the following series
20, 18, 15, 7, 8, 9, 4, 3, 10.
:

2. Calculate R,, D, and Par for the following distribution

70 60 50 40 30 20
Marks (more than): 80
:
7 18 40 40 63 70
No. of students
:

3. Determine the first quartile and the median from the following

- 40
Income (in Rs.): Below 30 30 40– 50 50-60 60-70 70-80 80 and above
No. of workers: 69 167 207 65 58 27 10

4. Calculate the quartiles and fourth decile from the following data

Marks: 5-10 10-15 15-20 20- 25 25-30 30 -35 35-40 40- 45


No. of students: 5 6 15 10 5 4 2 2

:
5. Find the mode, median, lst and 3rd quartile from the data

Wages: 0-10 10-20 20-30 30- 40 40 –50


No. of Workers: 22 35 46 35 20

ANSWERS
1. Q, =
5-5, Q
= 16-5, D, = 4, Peo = 10.

2. Q, = 34-56, D, = 36-09, Pas = 75


3. Q, = 34-89, Median = 43-16
4. Q = 15-417, Q = 25-75, D, = 17-87
5. Mode = 25, Median = 24-78, Q, = 15, Q = 34-429.
3 Measureof Dispersion

3.1. Introduction
The measure of central tendency (average) though gives us an idea about the population
from which the sample is taken but it fails to give more practical description while comparing
different samples of different population. It is clear from the following example.
The series of marks in Statistics (out of 50) of students of two different sections are
:
given below
SectionA: 18 19 22 24 25 25 26 28 31 32
Section B: 2 6 12 20 25 25 30 3 44 48
We notice that the average marks of students in each section is 25. But the quality
,
of
students in different sections isnot the same. The marks are spread out differently i.e. the
dispersion is different in each section.
Let us cho0se any student from section A and ask him to solve a problem on Statistics,
then we hope that he should be able to solve the problem as the standard of all students of
section A isreasonable and near the average. But if we give the samne problem to a student
of section B either he will solve it easily or he will not be able to solve it as there is a lot of
difference between the standard of the students in this section which is apparent fronm the
marks scored by them. Thus overall, student chosen from section A is more reliable than
the student chosen from section B.
If we plot the marks of students of section A and B on a straight line on the same
Scale, we see that the points corresponding to the marks of section A are close to each other
or branched together and those corresponding to the marks of section B are scattered or
spread out. We use the term dispersion to indicate this scattering or spreading out of the
difjereni values of a quantitative variable. Thus, we will say that the marks of section A have
a smaller dispersion about the mean as compared to the marks of section B.
From above discussion, it is clear that for comparison of different samples, measures of
central tendency are not sufficient to give complete information about a given data. Variability
or dispersion is another factor which is required to be studied under statistics. Like 'measures
of central tendeney' we want to have a single number to describe variability. This single
number is called a 'measure of dispersion'.
3.2. Measures of Dispersion
The extent to which numerical data tends to spread about un average value is
called
dispersion and the measures constructed to indicate the spread of the
aUerage in aset of obseruation's are
data about some
called measures of dispersion.
The measures of dispersion are :
1. Range 2. Quartile Deviation
3. Mean Deviation 4. Standard Deviation
1%
MATHEMATICS FOR E.CA

3.3. Range Though it


largest and the smullest observalion.
difference betwen the upon extreme values.
T'he range is the depends entirely
not reliable and
1S Casy to calculate,
yet it is
value (Sj
= Largest value (L)- Smallest
Symbolically. Range (R)

L,-S
Coefl. of
Range =
and L+S

(Q.D.)
3.4. Quartile Deviation
upper and the lower
quartiles i.e.,
difference between the
t is halfof the
Semi-Inter Quartile Range.
It is also called measure of Quartile Devjatic
variability, the
two or morc series for of Q.D. which is
given as
For comparing coefficient
purpose, we calculate
(Q.D.) will not work. Por this

=
Coef. of Q.D.
SOLVED EXAMPLES

samzle
rangeand quartile deviatior for the following
Example 1. Find the range,coeff. of
18, 14, 24, 20.
of observations : 9,
13, 23, 11, 15, 17, 25, L= 2;
observation in the given sample is 25 i.e.,
Solution. (i) We see that the largest
is 9 i.e., S=9.
and the smallest observation = 16.
Range =
L-S= 25 -9
16 = 0-471.
L-S 25-9
(ü) Coeff. of Range = 34 17
L+S 25+9
as:
After arrangement in ascending order we get observations
(üi)
24, 25.
9, 11, 13, 14, 15, 17, 18, 20,23,

Here n= 11

n+1 th item
Q, = value of
4
= value of 3rd item = 13.

Qa= value of
(3(n + 1)
th item
and

= value of 9th item = 23.

= 3-Q 23- 13
Quartile Deviation =5.
2 2
:
the following daia
Example 2. Caleulate the Rang, Q.D. and coefficient of QD. from

Clusses : 10 - 20 20- 30 30-40 40-50 | 50-60 60-70


5 15 10 4 2
Frequency :
MEASURE OF DISPERSION
3,3
1o
Solution, Here lower limit of snmallest class is
upper limit of highest class is 70
and = 70 - 10= 60.
Range
:
Calculation ofQ.D.
Classes Frequency (f) c.f
10 - 20
30 8
20- 5
30 - 40 15 23
40 - 50 10 33
50- 60 4 37
60 - 70. 2 39
N=39

=
N
Q, size of th item
4

39
th item =9-75th item.
4
Q, lies in group 30– 40
8
Here
=
l 30, h 10, f= 15, C=
Q, = l+
h(N -c= 30+(9-75
15
-8)
f\4
= 30
+x1·75=31·167 3
3N
Qg = size of th item

=
(3x39
size of th item
4
=
size of 29-25 th item
:. Q,lies in group 40 50 -
C =
Here l= 40, h= 10, f= 10, 23

Qa =
h(3N 10
(29-25 -23)
l+ 10
=.40 + 6-25 = 46-25
Q3 -Q1 46-25 -31-167 -=7-54.
Q.D. :
2 2
-
-Q. 4625 31-167
Also, Coeff. of Q.D. =
46- 25 +31-167
15-083 = 0-195.
77 1417
MATHEMATICS FOR BCA
33
follouing data
of QD. from the
Example 3. Calculate the Range, Q.D. and coefficient
rrlating to marks of 209 students. 7!- 80
81-9
61 - 70
Marks: 31 - 40 41 - 50 51- 6O
15
40 23
20
60 exclusive one by subtractig
:
No. of Students 40
inciusive series into
Solution. We shall first convert the given to upper limit of each Cass.
: irom lower limitof each ciass and adding 05
Calculations of Rangeand Q.D. c.f.
No. of students
Marks 40
40
305 – 405 100
60
40-5- 505 120
50-5 - 60-5
20
160
60-5 - 70-5
40
185
– 80-5 25
70-5 200
13
80-5 -g0-5
N = 200
of highest class 90-5 =
(2)Here upper limit
lower limitof smallest class = 30-5 - = 60.
and
Range =L-S= 90-5 30-5

Q, =Size of th itemn

= Size of 200 th item=Size of 50thitem


4
.. in group 40-5 –50-5
Q, lies
Here I =40-5, h =10, f= 60, C= 40
-) 10 x
10 –
40) = 40-5 + 10
=
40-5+(50 60
60
=
40-5 +1-67 = 42-17

Q4=Size of 3

4
th item
= Size of
324 th item=Size of 150th item

. Q lies in group 60-5-70-5


Here l = 60-5, h= 10, f= 40, C= 120
10
-c=605+ 40 (150 120)

10 =
+30
=60-5 60-5+7-5=68
40
ANEASURE
OF DISPERSION
3.5
(LAHO
Q.D. LIBRARY
Acc NoB:/RIS9)
G8-42.17 25-83
129BA
1 G8 --
42.17 A CANT
Cooficient of Q.),
Also,
684 42.17
25-83
0-234,
110-17

EXERCISE 3.1
:

I Calculate the range for the following sorics


15, 10, 11, 9, 13, 17, 20.
:

9. Find the range for the following distribution

Age(inyears): 15-- 17 17--19 19-- 21 21-- 23 23-- 25

No. of students: 9 8 2

:
8. Find the range from the following distribution

Marks: 0-10 10 -- 20 20 30 30 -40 40- 50

No. of students: 0 11 5 4
:

4, Find the quartile deviation for the following distribution

Marks: 4 5 7

No. of students: 10 11 12 13 5 12 7

:
5. Calculate the coefficient of Q.D. for the following data

Marks: 0-4 4
8 8-12 12- 14 14 -- 18 18- 20 20-25 25-30
No. of students: 10 12 18 7 5 4 6

6. Calculate range, Q.D. and coefficient of Q.D. from the data given below :

Wages: 1-5 6-10 11- 15 16- 20 21-25 26-30 31 -35


No. of 7
workers: 3 3 14 11 6 5

ANSWERS

1, 11
2. 10 3. 50
4. 2
5. 0-4721
Bo 35; Q, = 12-73; Q, = 24-61; Q,D. = 5-94; Cocf. of Q.D. = 0-3225.
MATHEMATICS FOR B.C

3.5. Mean Deviation (M.D.) mean of the numerical valueg


the arithmetic
statistical data is mean, mode or median) of the
Mean deiation ofa central alues (i.e.,
de:iation of each obserrations from the
distribution. by
series, mean deviation (M.D.) is given
For individual

where D=x, -a.


M.D. =
or we can write
M.D. =
a the central value or
is
x., are the values
of the variable and
Where l,, Ng..
average of the distribution.
For frequency
distribution

= =1 where D,= -a|


or we can write M.D.
M.D. =
i=1

where f is the frequency


of x(1 sisn).
Ef|x - a| or M.D, =
Ef|D|
Briefly M.D. N
N
=
where N Ef, and ifit
mean it is written as M.D. ()
mean deviation is calculated about
Remark. If
is calculated about median, it is
written as M.D. (median)
mean, then
If mean deviation is calculated about
for individual series M.D. (#)=

distribution MD. (F)= 2i; -|


and for frequency

if mean deviation is calculated about median, then


(median) = Z*; - median
for individual series M.D.

= Ef*;- median
and for frequency distribution, M.D. (median) N
ar
Note. When the values of variable are given, in the form of classes, then their respective midpoints
taken as the values of the variable.

3.6. Coefficient of M.D.


For the comparing two or more series for variability, the coefficient of M.D. is used ad
is dfined as
M.D.
Coeff, of M.D. =
Average
Remarks, 1. if
ID. is caleuiated about mean, then mean deviation is written as M.D.F)
and defined as
= MD. ()
Corfiient of I.D. ()
2, IfND. is calculated about median, then
M.D. (Mledian)
Coefieient of M.D. (Median) =
Median
en t s not mentiened Ghout whiehcentral ralue re hate to find M.D.. e choose #.D. about
Note.

SOL VED EXAMPLES

mean of the marks of 10 students of


Example l. nd the mean deviation about the
sctior A and B as given below:
10 12 13 15 20 21 27 30 35
Section A:
15 15 15 18 19 21 25 25
Section B: 15

Solution. We fñrst prepare the tables for section A andB.


190 B =
190 =
19 and mean marks for section 19
=

Mean marks for section A= 10


10
= 19.
Hence for both the sections,mean marks Le., I
Section A Section B
=x =
D, = (r,- ) Marks D,
|D,|
Marks
19) (x) (z =19)
(=

7 - 12 12 15 -4
9 13 -4 4
10 -9
7 15 -4 4
12 -7
-6 6 15 -4
13
15 -4 4 18 -1

20 1 1 19
21 2
21 2 2
3
27 22
11 25 6 6
30 11
16 25 6 6
35 16
= 190 E|D,| = 76 x, = 190 E |D,| = 34
Ex,

Mean deviation of section A =


E|D;|_76 = 7-6. (::n= 10]
10
MATHEMATICS FOR B.C.A. - 123
3.3

and mean deviation of section B E|D,|_ 34 = 3•4.


10
we can say that tha
mean deviation of section Bis less than that of section A, thus
Since
a dispersion about the mean as compared to section A.
marks of section B have smaller
:
Example 2. Find the mean deviation from A.M. of the following ata
9 10
3 5
10 7 4
6
f:
(M.D.):
Solution. Calculation ofmean deviation
= -
D

=:- x |D| |x | f|D|


f.x
3-9 3-9 11-7
2 3 6
3 6 18 - 2-9 2-9 17-4

-0-9 0-9 9-0


5 10 50
3-1 3-1 21-7
7 63
4-1 4-1 16-4
10 4 40
= 76-2
N=30 Efx = 177 Ef |D|
177
Now, A.M. ()= =5-9.
30
Ef|D| 76-2 = 2-54.
M.D. about mean = M.D. () =
N 30
Example 3. Find the mean deviation about the median of the following distribution.

6 12 18 24 30 36 42
f: 7 9 18 15 10 5

Solution. Calculation of mean deviation


:

f c.f. |D| = |x - med.| f|D|


6 4 4 18 72
12 7 11 12 84
18 9 20 6 54
24 18 38 0
30 15 53 6 90
36 10 63 12 120
42 5 68 18 90
N68 Ef |D 510

Here N= 68, which is even.


N
th item+ +-ilth item 34th item +35th item
Median
2 2
DISPERSION
MEASURE OF
39
Now. from above table, we observe that each of the items from 21st to 38th have value 24,
Thus value of both 34th and 35th items ís 24.
the
Median = 24
= Ef|D| 510
M.D. (median) 7-5.
N 68
Psample 4. Calculate the M.D. about mean and median and its coefficient for the following
:
distribution
Marks: 0- 10 10-20 20-30 30 -40 40 - 50
6
:

No. of students 5 15 16
= 10
Solution, (i) Mean Deviation about Mean: Let
A
25 and h=
Calculation of M.D. about mean:
Mid- No. of U; =
-
Marks values students D= |D|= f |D|
h
(x;) (A = 25, (7 = 27)
h= 10)

0- 10 -2 - 10 -22 22 110

10-20 15 8 -1 -8 - 12 12 96
20 - 30 25 15 0 -2 2 30
30– 40 35 16 1 16 8 128
40 - 50 45 6 2 12 18 18 108
=
N= 50 E fu,= 10 Ef|D| 472

= A+ h E fN = 25 + 10 %
10
= 27
Here
50

M.D.()= Ef|D| 472


50 =9-44
N
M.D. (#) 9-44 = 0-35.
Coeff. ofM.D. (T )=
27
(ii)Mean Deviation fromn Median:
Caleulation M.D. aboui median:
of

No. of Mid D = - med. ) |D| =


Marks students c.f. values (med. = 28) J - med. f|D|
(f) (x)
0- 10 5 5 -23 23 115
10- 20 13 15 - 13 13 104
20 - 30 15 28 25 -3 45
30 - 40 16 44 35 7 7 112
40 -50 6 17 17 102
50 45
N= 50 Ef |D|= 478
MATHEMATICS FOR BcA - 124
3.10

50 item= 25th item


tlh
Here N = 50. Median = size of

.. Median class is 20 -30


15, C= 13
l= 20, h= 10, f=
Mecian=
/4-|
10 x 12 28.
= 20
10
(25 - 13) = 20 + 15
15
478 =9-56
M.D.
(median)=|D| 50
M.D.(median) 9-56 0-34.
Coeff, ofM.D. (median) = median 28
EXERCISE 3.2

coefficient for the following


series
mean and its
mean deviation about
I. Find the 35
7, 10, 12, 13, 15,
20, 21, 27, 30,
:

following series
mean deviation from the median for the
2. Find the
50, 40, 60, 42, 51
34, 66, 30, 38, 44, :

10 rods in a shop are given below


3. The lengths (in cm) of 27-9, 30-2
52-3, 55-2, 72-9, 52-8, 79-0, 32-5, 15-2,
42-0,
(iü) Find the mean deviation from the mean also.
(i) Find M.D. (med) :

from the A.M. of the following data


mean deviation
4. Calculate the
11 13
5 7
3
2
2 10
f: :

mean (ii) from median for the following data


5. Find the mean deviation, (i) from

12 10 6
20 18 16 14
Marks:
14 1
:
2 4 18 27 25
No. of students
:
mean for the following distribution
6. Compute mean deviation about the

Class: 0-4 4-8 8-12 12-16 16– 20


5 2
Frequency : 4 6 8
:
7. Find mean deviation from median for the following distribution

Marks: 0-10 10 -20 20-30 30- 40 40- 50


No. of students: 5 10 20 5 10
DISTERSION D.11
WEASURE OF

8.
Calculate the meon doviation from nean, median nnd :coefficient of rnenn devíation #nd
cocíficient of median devintion from the following dnts
-
140 150 150- 1B0) 180-170 170- 18O 180 19) 190-2O9
Marka:
10 18
No. of sLudents:
mean deviation about mean and median of the folloiny
Caleulate the coefficient of
distributíon :

80 100 120 140 160 180 200 220 240


Wages (in Rs.): 20 40
1
workers: 3 13 43 102 175 220 204 139 25
No, of

ANSWERS

2. 8.7 3. (i) 16-44 (ii) 16-44


1.
7-6: 0.4
5. (i) 2.24 (ii) 2-24 6. 3-84 7. 9
4. 2-1257
8. 10-56, 0-062, 10-24, 0-059 9. 0-2285; 0-2333.

3.7. Limitations of Mean Deviation


a case of a very high
The median is not a representative central tendency in series in
degree of variability. "Thus, the mean deviation about median calculated
for such series
cannot be fully relied upon. Also, the sum of the deviations from the
mean (-ve signs
mean deviation about
ignored) is more than the sum of deviations fromn the median. So, the
mean is not very scientific. "Therefore, in many cases, mean deviation gives unsatisfactory
on the basis of absolute values of
results. Also, the calculation of mean deviatipn is made
we
the deviations and as such they cannot be given any further algebraic treatment. Thus,
cases stundard
require some other measure of dispersion to overcome the problem, In such
deviation is found, very useful which shall be studying in the following article.

3.8. Standard Deviation (S.D.)


The standard deviation of u statistical data is defined as the positive square root of the
urithmetic mean of the square of deviations of items from the arithmetic mean of he series
under consideration. It is generally denoted by a (read as sigma). It is also known as root
mean square deviation.
For an individual series:
n
(*, -)?
S.D. (o) = |i=1 or briefly o = where d, = x;- x
n

where x, xgg ., , are the values of variable and is the arithmetic mean.
MATHEMATICS FOR B.C,A. 123

3.12

For a frequency distribution :

where d, =x
=
or briefly o= N
S.D. (o)

where f is the frequency of x, (1 sisn). is denoted


varianceand
square of the standard deuiation is called
VARIANCE. The
by o. the variable are given in the form
distribution, when the values of
Remark. In a freguency are taken as the values of the variabla
respective mid-points
of classes, then their COEFFICIENT OF VARIATION
STANDARD DEVIATION AND
corresponding
3.9. COEFFICIENT OF variability, we calculate the
two or more series for
For comparing
relative measure. as
coefficient of standard deviation and is defined
measure is known as
This
S.D.
=
Coeff. of S.D.

Coefficient of variation (C.V.) is


defined as

C.V. = x 100

of comparing coeff. of S.D.


advisable to compare C.V. instead
Note. In practical problems, it is
Deviation
3.10. Methods to Calculate the Standard
:
3.10.1. Direct Method

=
Ef(;-) .(1)
We know that o,, N

E(6-2x,i+r)
N

|1 2-2x- 2f +
=
N
-
fx-27.7+7 and N =
VN N
1

-?
2

...(2)
VN N
D9T'ERSJCON
MEASUIRE OF

3.10,2. Change of Orign (9hort-cut Method)


n
Uben the mean of distribution ÍA decimal fractlon, the caleulation work hecme8
cnsCA We Lnke IAAUMed
mean, whlch in nny arbitrary numher A, and find
So Auch
tedious, in
the doviation from
this men,
A L.e, A is the neyy
= x, – A i.e., d, aro dovintiona of x, from an arbitrary numher
Lot d,
orgin.

N N N

d, - d =
d,-(7-A) x,-A-+A
= =
x,- 7 ...(3)

Now N
N
...14)
i.e.,
as
Using (2), equation (4) can be written

N N

Result (4) shows that standard deviation remains unchanged by the change of origin.
Remark. For individual series, the above formula takes the form
2
S.D. = ...(5)

3.10.3. Changeof Origin and Change of Scale (Step Deviation Method)


- A

Let where A and h are arbitrary constants.


h
= ...
*,-A hu, x,=A + hu, (6)
-
=A h.u
+
X=x,-(A +h.u)
x, – x = A + hu,- (A + hu) (Using (6))

* -* = h(u, -) ..(7)
1
Now, [Using (7))

1
(:: h is constant)
=
h²o,?
-
3.14 MATHEMATICS FOR B.C.A. ))

..(8)
o, =h, N N

a
and h are constants,
of x, (1 sisn) and u,=
-(1sisn);
where f is the frequency
2

S.D. (o) =h
Ef"_(2f"
N N
Thus
scale
independent of change of origin but not change of
is
Equation (8)shows that S.D. of class-intervals.
may be taken as the width
Note. ln numerical problens
h
SOLVED EXAMPLES

following data
Example 1. Compute the standard deviation for the
7
4 5
2
16 10
6 12 18 26
:

Solution. Calculation of standard deviation (S.D.)

f
-3 54
1 6 6
-2 4 48
12 24
1 18
18 54 -1
0
4 26 104
80 1 1 16
5 16
60 2 4 40
6 10
7 8 56 72
=
= 384 Ef; (x;- 248
N= 96 2;*,
Ef;x 384
=4
llere N 96

|1
Now, VN

248 =
96
16 (nearly).
DISPERSION 3.15
MEAŠURE
Find the standard deviation of the first
n
natural numbers.
Example 2, formula
Solution.The S.D. is given by the
(Since frequency of each variate is onel

n(n + 1)
3 +
... + n =
Ex, = 1+2+ 2
+ 1)
lere ... + n(n + 1)(2n
= 12+ 2 +
32 + n2=
Ex? 6

n(n + 1)
2
(n 1)(2n+1)
+ (n +1)
n(n +1)(2n +1)
4
6n 2-n

(2n + 1)

6 4

(n + 1(n - 1)
12 :

variance and standard deviation for the following data


Example 3. Find the
17 20 24 32
4 11
9 5 4 3
3 5
:

Solution. Calculation of variance and


standard deviation
-
d, x,-
=
d' = (x, ?
4 12 - 10 100 300
36 180
5 40
-3 81
11 9 99
3 45
17 5 85
6 36 144
20 4 80
10 100 300
24 3 72
18 324 324
32 1 32
= 1374
N=30 E fx, = 420 Efd?
2f;x; 420 = 14 [: N= 30)
Now,
N 30
1 = x;
Variance (o) =
N
f d', where d, -7
i=1

1
×
1374=45-8.
30
and S.D. (o)= /45-8 =6-77.
MATHEMATICS FOR B.C.A.
- 1%
3,]6

the following
ISxample 4. Calculate the mean, variance and standard deviation for
distribution :

60 -70 70 - 80) 80 -90 90- 100


Class : 30
--
40 40-50 60 -60
8 3
7 12 15
Frequency:
of mean, variance and S.D.:
Solution. Calculation
Class Mid-value Frequency
(x = 62)
(x) - 27 729 2187
30 - 40 3 105
35
- 17 289 2023
40 - 50 7 315
45 588
660 -7 49
50 - 60 55 12
135
15 975
60 -70 65
13 169 1352
600
70 – S0 75 1587
255 23 529
S0 - 90 85 3
1089 2178
190 33
90 - 100 95 2

= 10050
N= 50 = 3100

= Efx 3100 =
62.
Mean ()
50
Ef; d 10050 =
Variance (o?) = 201
N 50

and
/201 = 14-17.
S.D. (o) = /var, =

mean, standard deviation and coefficient of variati


Example 5. Find the arithmetic
:
(C.V)from the followingdata
10 11 12 13 14 15 16
Size of item:
2 7 11 15 10 4
Frequency :

Solution. Let A = 13. We may now prepare the following table :

Size of the item (e) Frequency (f) d, =


x- A d
f;d,
(A = 13)
10 2 -3 9 -6 18
11 -2 4 - 14 28
12 11 -1 1 - 11 11
13 15 0 0
14 10 1 1 10 10
15 4 16
2 4 8
16
3 9 3

£fd, =-
N 50 = 92
10 fd
DISPERSION
MEASURE OF
3.17

L{di
Now,
A.M. ( ) A +
N
13 +| -10
50
12-8.

2
Efa?
S.D. =
N

2
92 -10
=50 50

=
J1:84 -0-04 =f1.80 =1-342.

S.D.
C.V. = x 100

x100 = 10-482%.

Example 6. Calculate standard deviation and its coefficient of variation from the following
data:
Wages upto(in Rs.) : 10 20 30 40 50 60 70 80

No. of persons : 12 30 65 107 157 202 220 230

Solution. Let A = 35 and h = 10. We may now prepare the following table:

Wages No.of persons f) Mid-value (x) u,= 4-35


10

12 5 -3 -36 108
0-10
10-20 18 15 -2 -36 72

20- 30 35 25 -1 -35 35

30 – 40 42 35 / 0 0

40– 50 50 45 1 50 50

50-60 45 55 2 90 180

60- 70 18 65 3 54 162

70- 80 10 75 4 40 160
N
230 Efu,=127| fu?= 767
127 x
= A +
Žf4h =
35 + 10 = 40-52:
N 230
2
|767 -(0-B52)2
V
N 230

--
= 10./3-335 0-3047 = T0/3-0303 = 10(1-741) = 17-41.
082 sereug arp aa
mm) of the heads of 07
EXAMnle 7. 7}e soNHngsgnte ofthe diametrs in
deriatioM.
Eitnbon Ciruletr the standard 45
- 7
4244
19 23 21 27
form
it into an exclusive
in incusive foT. Converting
Solution, Herp the nven data isand ndding 0 5 to upper limits, we have the clagses
a
y
subiracting 05- fn .lower limits
325-R56, 335 3S5, and so on.

following table:
Wr may now prNparr the
A
Frequency fpd,
Mid-values
hameter (A 40: h
=
3)

-34 68
325 - 35-5 34 17
- 19 19
S3-5 - S8 5 37 19
0
3S5 - 41-5 40 23
21 21
43 21
41$-44-5 54 108
2
445 - 475 46 27
= 216
N= 107 Efd, = 22 fd
2
216 22
S. D.(o) = h -
(E{d = 3x
N 107 107

=
3x /2-0187 - 0-042
= x 1-41 = 4-23.
3x/1-976 =3
two classes. State which
Example 8. Following is the table giving weight of students of
ciass is more variable ?

Weight (in Kg) Class A Class B

20 - 3O 7 5

30 - 40 10 9
40 - 50 20 21

50 - 6O 18 15

60 - 70 7 6

N=62 N= 56
pISPERSION
MEASURE OF 3,19
:
clasa A
colution, Calculation ofC.V. for
Let 45 and h= 10
A

Weight
No. of students (f) Mid-values (x) d, =
10

7 26 -2 - 14 28
20- 30 35 -1 - 10 10
30 40- 10
0
20 46
40- 50 1 18 18
- 60 18 55
50
7 65 2 14 28
60- 70
= 84
N= 62 Efd, = 8 Efap

2
= 10 ×, 84
S.D. =
V62

= 10 x = 11-57
1-355 –0-0167
= 45 +
xh
N
8 x
co|
= 45 10 = 46-29
62
S.D.
C.V. for class A |x 100 = 25 %

= 45 and 10
Calculation of C. V.

for class B: Let A h=


~45
Weight No. of
students (f) Mid-values «) d,
10

20- 30 25 -2 10 20
30 - 40 35 -1 -9
40-50 21 45 0
50- 60 15 55 1 15 15
-60-70 6 65 2 12 24
= 8 = 68
N= 56 Ef.d, Ef.d
co 2
Now, S.D. = h Ef;d; = 10x 68
N N V56 56
- 10x 1214 -0-0204 = 10 x
1-1936 = 10-92

= 45 +
xh
=
45 + x 10= 46-428
N 56
i2%
MAT 1MATICS POM 9CA,
3.20

-
C.V. for class
B
=x 100 10-92
46.428
1o0)
Now CV. ofA> C.V. of B

Class A is mnore variable. ure given below. Whi:h Gf


Example 9. The scores of tiwo batsmen A and fo B
20 inLninga
Che twO May be regarded as the mre consistent butsman ?

50 51 62 53
Score:
A:
No. of Innings:
B: 2

C.V. for batsman A


:
Let A = 53.
Solution, Calculation of
Score No. of Innings d; = x, A -
(A = 53)
(z;)
3
50
51
52
53 4
3
54 24
6 2 12
55 27
56 3
4 12 48
57
20 Ef;d, = 33 E
fal2= 111
N=
= A + = 33
Here Mean () N
53 + 54 -65
20

and S.D. () = 111 33 = /5-55 - 2-72 = 1-6815


N V 20
c.v. for A
-(]* 100 =
1-6815 100 = 3-0768%
54.65
Calculation of C.
V. for batsnan B:
Score No. of Innings d; = x; A - fid
(x,) (A = 53)
50 1 -3 3 9
51 2 -2 4
52 2 2
53
54 3 3 3

55 16
56 2 3 18
57 4
N=20 Ef d,=8 Efd,2 = 56
DISPERSION
MEASURE OF 3.21

JJere Mean (T) = A 4


= 53 + = 53•4
20

And
S.D. (o) = (Efd |56 =
J2-8-0.16 = 1-6248
N 20

C.7. for B = x
100 = (1-6248 x 100 = 3-0427%
53.4
C.V. for A
>0.V. for B
Hence batsman B in more consistent.
311. S.D. of Combined Group
Let G,, G,,..., o, be S.D. of m groups containing n,, ngy.,n,
items respectively. Let
m
g
i.E,... be their respective A.M., be the combined A.M. and be the combined S.D.

j=1
m

j=1

.., m
We define d,=
ï - ž, j= 1,2,
. S.D. of the combined group is given by
m m

j=1 j=1 j=1


m
M

j =l

3.12. Relation Between Measures of Dispersion


The following relations also hold in the case of frequency distribution.

(a) 2 S.D. 4
Q.D. (6) M.D. s 5
S.D.

Example 10. The following data gives the A.M. and S.D. ofthree subgroups. Calculate the
A.M. and
S.D. of the combined group.

Subgroup No. of men Average wages (in Rs.) S.D. (in Rs.)
A 150 20 2

B 150 10 3
C 200 12-5 4
101
MATHEMATICS FOR B.C.A.
3.22

Solution. Here X =20,


, =
10, 12-5

= l50, no = 150, n, = 200


Og = Oz = 4
o, = 2, 3,

Let be the A,M. of


thc combined group

+ x (12·5)
150 x 20 + 150x 10 200
150 + 150 + 200

3000 + 1500 + 2500 7000 = 14


500 500

Now,
d
=
i -x = 20 - 14 = 6
d, = I, - x= 10 - 14 =-4
do = T -= 12-5- 14 =-1-5
group
Let o be the S.D. of the combined
nyo 2 +ngo22 + ngG3 + n,d, + n,d,+ngdg 2

n, (o, + + ng(a + d
)+ ng (o d)
+

d,')

150(4-+36)+ 150 (9 + 16) 200 (16 +


+ 2- 25)

150 + 150 + 200


13400
=5-177.
V
500
as 40 and 5-1 respectively.
Example 11. The A.M., and S.D, of 100 items was recorded as 50. Find the
Later on it was discovered that one observation 40 was wrongly copied down
correct S.D.
Solution. No. of items = 100
Incorrect Mean () =40
Incorrect S.D. =5-1
Incorrect item = 50
Correct item = 40
Incorrect Ex
Now 40 =
100
Incorrect Ex = 4000
SASUNE OF DISTERSION 323

50+ 40 = 3990

Correct L 4000
3990
Correct Mean =399
100

S.D. =
Now

51 =
Incorrect Ex? .- (40)
100
we have
Squaring on both sides,
Incorrect E? - 1600
26-01 =
100
E = 162601
Incorrect
-
Correct =162601 (50)+ (40)= 161701
161701 (39-9)*
Correct S.D. = V 100

J1617-01 1592-01 =5

Example 12. The mean of 5 observations is 4 and variance


is 5-2. If three of the fve
two.
observations are 1, 2and 6, find the other
a b.
Solution. Let the missing observations be and
Given n =5. F =4, o'= 5-2
We have Ex=nxT= 5x4=20
n
20 ...(1)
Now, =1+2+6+a+6=
Ex
...(2)
a +b = 20
-9 = 11

We have
- (T)2

52 =
- (4)2
5.
Or Dy2 - (5-2 + 16) 5 = 106
= 106 (Using (1)}
j 12 + 22 + 6+ a'+b
a+ b2 =65
Now, (a +b)
=
a2+b2 + 2ab
i.e., 121 = 65 + 2ab
ab = 28 ...(3)

Solving (2) and (3) for a and b, we get


a =4,b=7.
MATHEMATICS FOR BC.A.
- 13:
3.24
EXERCISE 3.3

1. Find S.D. and C.V. for the data


:
4, 6, 10, 12, 18. :
coefficient of variation for the
following data
deviation and
2. Calculate the standard
15 25 35 45 55

18 27 20 17 6
12
method
devíation for the following data using short-cut
3. Findthe mean and standard
65 66 67 68
60 61 62 63 64
12 10 5
2 1 12 29 25
|6 distribution.
Calculate the standard deviatíon for the following frequency
4.
4-8 8-12 12-16
Class interval: 0–4
2 1
Frequency : 4
:

5. Calculate the
mean, median and standard deviation of following distribution

40-50 50-60 60-70 70–80 80-90


Age (in years): 30-40 20-30
153 140 51 2
No. of members: 61 132
:

6. Calculate the S.D. and C.V. for the following frequency distribution
36– 40
Class
:
4-8 8-12 12- 16 16– 20 20- 24 24- 28 | 28–3232-36
13 16 14 14 17 6 4
Frequency : 11

the standard deviation of the


7. In a study on patients, the following data was obtained. Find
:
data
Age (in years): 10-19 20 - 29 30-39 40– 49 50- 59 60- 69 70–79 80- 89
No. of cases: 1 10 17 38 3

:
8. Calculate the C.V. fur the following data

Class interval: 60-70 50-60 40- 50 30 – 40 20- 30 10 -20


Frequency : 3 6 10 12 15 6

9. Find which of the following batsmen is more consistent in scoring

Batsman A: 5 7 16 27 39 53 56 61 80 101 105


Batsman B: 4 16 21 41 43 57 79 83 90 95

10. The following are the scores made by Dhoni and Yuvraj in a series of innings :

Dhoni 12 115 73 7 -19 119 36 84 29


Yuvraj :
47 12 76 42 51 37 48 13 0

(i) Who is a better rungetter ? (ii) Who is more consistent ?


DISPERSION
EASURE OF 3.25
a
Goals soored by two teams and in football season were as followvs
A B

11. :

No. of goals scored in a match No. of Mlatches


A B
27
9 5
2
S 5

1 27
ind out which team is more consistent,
12. The following data gives the A.M. and S.D. of three sub groups. Calculate the A.M, and
sD. of the combined group.
Sub group No. of men Average wages (in Rs.) S.D. (in Rs.)
A 50 61 8
B 100 70
120 80-5 10
13. The mean and S.D. of 20 items is found to be 10 and 2 respectively, It was later discovered
that one item 12 was wrongly recorded as 8. Calculate the correct mean and S.D.
14. The mean of 5 observations is 4-4 and the variance is 8-24. If three of the five observations
are 1, 2 and 6, find the other two.
15. An original frequency distribution with A.M.= 11, variance = 9.9 was lost but the following
table derived from it was found. Construct the original table

Step deviation -2 -1 0 1 2
Frequency 1 6 7 4 2

16. (a) If the standard deviation of a series is 7-5, find the most likely value of the mean
deviation.
(6) A group of 100 selected students is with average height 168-8 cm and C.V. 3-2% .What
is standard deviation of their height.

ANSWERS

14-10; 50-35% 3. 64; 1-69


1. 4-898, 48-989% 2.
4. 3-2659 5. 54-7: 54-9, 119
6. S.D, = 9.1146 : C.V., = 45-573% 7. 11-28 8. 38-708%
= 67-07%: 69-51%; A is more consistent.
9. C.V. for A
C.V. for
B=
10. (i) A.M. for Dhoni (50) > A.M. for Yuvraj (33)
.:.
Dhoni is better rungetter
(iü)C.v. for Dhoni = 83-661%: C.V. for Yuvraj = 70-820%
.. Yuvraj is more consistent.
MATHEMATICS FOR B.C.A.
3.26

11. C.V. for team A = 127-83: C.V. for team B = 36-05

Team B is more consistent.


12. 73; 11-8868
13. Correct I = 10-2 : Correct S.D. = 1-9899

14. 9, 4

9.5-12·5 12-5- 15.5 15-5-- 18-5


Class :
3.5-6-5 65-9.5
15. 7 4 2
Frequency : 1 6

16. (a) 6 (b) 5.4016 cm.

You might also like